Monday, August 20, 2018

Guest Post: Tam Hunt questions Carlo Rovelli about the Nature of Time

Tam Hunt.
[Tam Hunt, photo on the right, is a renewable energy lawyer in Hawaii and an “affiliate” in the Department of Psychological and Brain Sciences at UC Santa Barbara. (Scare quotes his, not mine, make of this what you wish.) He has also published some papers about philosophy and likes to interview physicists. The below is an email interview he conducted with Carlo Rovelli about the Nature of Time. Carlo Rovelli is director of the quantum gravity group at Marseille University in France and author of several popular science books.]

TH:Let me start by asking why discussions about the nature of time should matter to the layperson?

CR: There is no reason it “should” matter. People have the right to be ignorant, if they wish to be. But many people prefer not to be ignorant. Should the fact that the Earth is not flat matter for normal people? Well, the fact that Earth is a sphere does not matter during most of our daily lives, but we like to know.

TH: Are there real-world impacts with respect to the nature of time that we should be concerned with?

CR: There is already technology that has been strongly impacted by the strange nature of time: the GPS in our cars and telephones, for instance.
Carlo Rovelli.

TH: What inspired you to make physics and the examination of the nature of time a major focus of your life's work?

CR: My work on quantum gravity has brought me to study time. It turns out that in order to solve the problem of quantum gravity, namely understanding the quantum aspects of gravity, we have to reconsider the nature of space and time. But I have always been curious about the elementary structure of reality, since my adolescence. So, I have probably been fascinated by the problem of quantum gravity precisely because it required rethinking the nature of space and time.

TH: Your work and your new book continue and extend the view that the apparent passage of time is largely an illusion because there is no passage of time at the fundamental level of reality. Your new book is beautifully and clearly written -- even lyrical at times -- and you argue that the world described by modern physics is a “windswept landscape almost devoid of all trace of temporality.” (P. 11). How does this view of time pass the “common sense” test since everywhere we look in our normal waking consciousness there is nothing but a passage of time from moment to moment to moment?

CR: Thanks. No, I do not argue that the passage of time is an illusion. “Illusion” may be a misleading word. It makes it seem that there is something wrong about our common-sense views on time. There is nothing wrong with it. What is wrong is to think that this view must hold for the entire universe, or that it is valid at all scales and in all situations. It is like the flat Earth: Earth is almost perfectly flat at the scale of most of our daily life, so, there is nothing wrong in considering it flat when we build a house, say. But on larger scales the Earth just happens not to be flat. So with time: as soon as we look a bit farther than our myopic eyes allow, we see that it works differently from what we thought.

This view passes the “common sense” test in the same way in which the fact that the Earth rotates passes the “common sense” view that the Earth does not move and the Sun moves down at sunset. That is, “common sense” is often wrong. What we experience in our “normal waking consciousness” is not the elementary structure of reality: it is a complex construction that depends on the physics of the world but also on the functioning of our brain. We have difficulty in disentangling one from the other.

“Time” is an example of this confusion; we mistake for an elementary fact about physics what is really a complex construct due to our brain. It is a bit like colors: we see the world in combinations of three basic colors. If we question physics as to why the colors we experience are combination of three basic colors, we do not find any answer. The explanation is not in physics, it is in biology: we have three kinds of receptors in our eyes, sensible to three and only three frequency windows, out of the infinite possibilities. If we think that the three-dimensional structure of colors is a feature of reality external to us, we confuse ourselves.

There is something similar with time. Our “common sense” feeling of the passage of time is more about ourselves than the physical nature of the external world. It regards both, of course, but in a complex, stratified manner. Common sense should not be taken at face value, if we want to understand the world.

TH: But is the flat Earth example, or similar examples of perspectival truth, applicable here? It seems to me that this kind of perspectival view of truth (that the Earth seems flat at the human scale but is clearly spherical when we zoom out to a larger perspective) isn’t the case with the nature of time because no matter what scale/perspective we use to examine time there is always a progression of time from now to now to now. When we look at the astronomical scale there is always a progression of time. When we look at the microscopic scale there is always a progression of time.

CR: What indicates that our intuition of time is wrong is not microscopes or telescopes. It’s clocks. Just take two identical clocks indicating the same time and move them around. When they meet again, if they are sufficiently precise, they do not indicate the same time anymore. This demolishes a piece of our intuition of time: time does not pass at the same “rate” for all the clocks. Other aspects of our common-sense intuition of time are demolished by other physics observations.

TH: In the quote from your book I mentioned above, what are the “traces” of temporality that are still left over in the windswept landscape “almost devoid of all traces of temporality,” a “world without time,” that has been created by modern physics?

CR: Change. It is important not to confuse “time” and “change.” We tend to confuse these two important notions because in our experience we can merge them: we can order all the change we experience along a universal one-dimensional oriented line that we call “time.” But change is far more general than time. We can have “change,” namely “happenings,” without any possibility of ordering sequences of these happenings along a single time variable.

There is a mistaken idea that it is impossible to describe or to conceive change unless there exists a single flowing time variable. But this is wrong. The world is change, but it is not [fundamentally] ordered along a single timeline. Often people fall into the mistake that a world without time is a world without change: a sort of frozen eternal immobility. It is in fact the opposite: a frozen eternal immobility would be a world where nothing changes and time passes. Reality is the contrary: change is ubiquitous but if we try to order change by labeling happenings with a time variable, we find that, contrary to intuition, we can do this only locally, not globally.

TH: Isn’t there a contradiction in your language when you suggest that the common-sense notion of the passage of time, at the human level, is not actually an illusion (just a part of the larger whole), but that in actuality we live in a “world without time”? That is, if time is fundamentally an illusion isn’t it still an illusion at the human scale?

CR: What I say is not “we live in a world without time.” What I say is “we live in a world without time at the fundamental level.” There is no time in the basic laws of physics. This does not imply that there is no time in our daily life. There are no cats in the fundamental equations of the world, but there are cats in my neighborhood. Nice ones. The mistake is not using the notion of time [at our human scale]. It is to assume that this notion is universal, that it is a basic structure of reality. There are no micro-cats at the Planck scale, and there is no time at the Planck scale.

TH: You argue that time emerges: “Somehow, our time must emerge around us, at least for us and at our scale.” As such, how do you reconcile the notion of emergence of time itself with the fact that the definition of emergence necessarily includes change over time? That is, how is it coherent to argue that time itself emerges over time?

CR: The notion of “emergence” does not always include change over time. For instance we say that if you look at how humans are distributed on the surface of the Earth, there are some general patterns that “emerge” by looking at a very large scale. You do not see them at the small scale, you see them looking at the large scale. Here “emergence” is related to the scale at which something is described. Many concepts we use in science emerge at some scale. They have no meaning at smaller scales.

TH: But this kind of scale emergence is a function solely of an outside conscious observer, in time, making an observation (in time) after contemplating new data. So aren’t we still confronted with the problem of explaining how time emerges in time?

CR: There is no external observer in the universe, but there are internal observers that interact with one another. In the course of this interaction, the temporal structure that they ascribe to the rest may differ. I think that you are constantly misunderstanding the argument of my book, because you are not paying attention to the main point: the book does not deny the reality of change: it simply confronts the fact that the full complexity of the time of our experience does not extend to the entire reality. Read the book again!

TH: I agree that common sense can be a faulty guide to the nature of reality but isn’t there also a risk of unmooring ourselves from empiricism when we allow largely mathematical arguments to dictate our views on the nature of reality?

CR: It is not “largely mathematical arguments” that tell us that our common sense idea of time is wrong. It is simple brute facts. Just separate two accurate clocks and bring them back together and this shows that our intuition about time is wrong. When the GPS global positioning system was first mounted, some people doubted the “delicate mathematical arguments” indicating that time on the GPS satellites runs faster than at sea level: the result was that the GPS did not work [when it was first set up]. A brute fact. We have direct facts of evidence against the common-sense notion of time.

Empiricism does not mean taking what we see with the naked eye as the ultimate reality. If it was so, we would not believe that there are atoms or galaxies, or the planet Uranus. Empiricism is to take seriously the delicate experience we gather with accurate instruments. The idea that we risk unmooring “ourselves from empiricism when we allow largely mathematical arguments to dictate our views on the nature of reality” is the same argument used against Galileo when we observed with the telescope, or used by Mach to argue against the real existence of atoms. Empiricism is to base our knowledge of reality on experience, and experience includes looking into a telescope, looking into an electronic microscope, where we actually can see the atoms, and reading accurate clocks. That is, using instruments.

TH: I’m using “empiricism” a little differently than you are here; I’m using the term to refer to all methods of data gathering, whether directly with our senses or indirectly with instruments (but still mediated through our senses because ultimately all data comes through our human senses). So what I’m getting at is that human direct experience, and the constant passage of time in our experience, is as much data as are data from experiments like the 1971 Hafele-Keating experiment using clocks traveling opposite directions on airplanes circling the globe. And we cannot discount either category of experience. Does this clarification of “empiricism” change your response at all?

CR: We do not discount any category of experience. There is no contradiction between the complex structure of time and our simple human experience of it. The contradiction appears only if we extrapolate our experience and assume it captures a universal aspect of reality. In our daily experience, the Earth is flat and we take it to be flat when we build a house or plan a city; there is no contradiction between this and the round Earth. The contradiction comes if we extrapolate our common-sense view of the flat Earth beyond the small region where it works well. So, we are not discounting our daily experience of time, we are just understanding that it is an approximation to a more complicated reality.

TH: There have been, since Lorentz developed his version of relativity, which Einstein adapted into his Special Theory of Relativity in 1905, interpretations of relativity that don’t render time an illusion. Isn’t the Lorentz interpretation still valid since it’s empirically equivalent to Special Relativity?

CR: I think you refer here to the so called neo-Lorentzian interpretations of Special Relativity. There is a similar case in the history of science: after Copernicus developed his systems in which all planets turn around the Sun and the Earth moves, there were objections similar to those you mention: “the delicate mathematical arguments” of Copernicus cannot weight as much as our direct experience that the Earth does not move.

So, Tycho Brahe developed his own system, where the Earth is at the center of the universe and does not move, the Sun goes around the Earth and all the other planets rotate around the Sun. Nice, but totally useless for science and for understanding the world: a contorted and useless attempt to save the common sense-view of a motionless Earth, in the face of overwhelming opposite evidence.

If Tycho had his way, science would not have developed. The neo-Lorentzian interpretations of Special Relativity do the same. They hang on to the wrong extrapolation of a piece of common sense.

There is an even better example: the Moon and the Sun in the sky are clearly small. When in antiquity astronomers like Aristarchus come out with an estimate of the size of the Moon and the Sun, it was a surprise, because it turned out that the Moon is big and the Sun even bigger than the Earth itself. This was definitely the result of “largely mathematical arguments.” Indeed it was a delicate calculation using geometry, based on angles under which we see these objects. Would you say that the fact that the Sun is larger than the Earth should not be believed because it is based on a “largely mathematical argument“ and contradicts our direct experience?

TH: But in terms of alternative interpretations of the Lorentz transformations, shouldn’t we view these alternatives, if they’re empirically equivalent as they are, in the same light as the various different interpretations of quantum theory (Copenhagen, Many Worlds, Bohmian, etc.)? All physics theories have two elements: 1) the mathematical formalisms; 2) an interpretive structure that maps those formalisms onto the real world. In the case of alternatives to Special Relativity, some have argued that we don’t need to adopt the Einstein interpretation of the formalisms (the Lorentz transformations) in order to use those formalisms. And since Lorentz’s version of relativity and Einstein’s Special Relativity are thought to be empirically equivalent, doesn’t a choice between these interpretations come down to a question of aesthetics and other considerations like explanatory power?

CR: It is not just a question of aesthetics, because science is not static, it is dynamic. Science is not just models. It is a true continuous process of better understanding reality. A better version of a theory is fertile: it takes us ahead; a bad version takes no part. The Lorentzian interpretation of special relativity assumes the existence of entities that are unobservable and undetectable (a preferred frame). It is contorted, implausible, and in fact it has been very sterile.

On the other hand, realizing that the geometrical structure of spacetime is altered has led to general relativity, to the prediction of black holes, gravitational waves, the expansion of the universe. Science is not just mathematical models and numerical predictions: it is developing increasingly effective conceptual tools for making sense and better understanding the world. When Copernicus, Galileo and Newton realized that the Earth is a celestial body like the ones we see in the sky, they did not just give us a better mathematical model for more accurate predictions: they understood that man can walk on the moon. And man did.

TH: But doesn’t the “inertial frame” that is the core of Einstein’s Special Relativity (instead of Lorentz’s preferred frame) constitute worse “sins”? As Einstein himself states in his 1938 book The Evolution of Physics, inertial frames don’t actually exist because there are always interfering forces; moreover, inertial frames are defined tautologically (p. 221). Einstein’s solution, once he accepted these issues, was to create the general theory of relativity and avoid focusing on fictional inertial frames. We also have the cosmic frame formed by the Cosmic Microwave Background that is a very good candidate for a universal preferred frame now, which wasn’t known in Einstein’s time. When we add the numerous difficulties that the Einstein view of time results in (stemming from special not general relativity), the problems in explaining the human experience of time, etc., might it be the case that the sins of Lorentzian relativity are outweighed by Special Relativity’s sins?

CR: I do not know what you are talking about. Special Relativity works perfectly well, is very heavily empirically supported, there are no contradictions with it in its domain of validity, and has no internal inconsistency whatsoever. If you cannot digest it, you should simply study more physics.

TH: You argue that “the temporal structure of the world is not that of presentism,” (p. 145) but isn’t there still substantial space in the scientific and philosophical debate for “presentism,” given different possible interpretations of the relevant data?

CR: There is a tiny minority of thinkers who try to hold on to presentism, in the contemporary debate about time. I myself think that presentism is de facto dead.

TH: I’m surprised you state this degree of certainty here when in your book you acknowledge that the nature of time is one of physics’ last remaining large questions. Andrew Jaffe, in a review of your book for Nature, writes that the issues you discuss “are very much alive in modern physics.”

CR: The debate on the nature of time is very much alive, but it is not a single debate about a single issue, it is a constellation of different issues, and presentism is just a rather small side of it. Examples are the question of the source of the low initial entropy, the source of our sense of flow, the relation between causality and entropy. The non-viability of presentism is accepted by almost all relativists.

TH: Physicist Lee Smolin (another loop quantum gravity theorist, as you know) argued views quite different than yours, in his book, Time Reborn, for example. In an interview with Smolin I did in 2013, he stated that “the experience we have of time flowing from moment into moment is not an illusion but one of the deepest clues we have as to the nature of reality.” Is Smolin part of the tiny minority you refer to?

CR: Yes, he is. Lee Smolin is a dear friend for me. We have collaborated repeatedly in the past. He is a very creative scientists and I have much respect of his ideas. But we disagree on this. And he is definitely in the minority on this issue.

TH: I’ve also been influenced by Nobel Prize winner Ilya Prigogine’s work and particularly his 1997 book, The End of Certainty: Time, Chaos and the New Laws of Nature, which opposes the eternalist view of time as well as reversibility in physics. Prigogine states in his book that reversible physics and the notion of time as an illusion are “impossible for me to accept” He argues that whereas many theories of modern physics include a reversible t term, this is an empirical mistake because in reality the vast majority of physical processes are irreversible. How do you respond to Prigogine and his colleagues’ arguments that physics theories should be modified to include irreversibility?

CR: That he is wrong, if this is what he writes. There is no contradiction between the reversibility of the laws that we have and the irreversibility of the phenomena. All phenomena we see follow the laws we have, as far as we can see. The surprise is that these laws allow also other phenomena that we do not see. So, something may be missing in our understanding --and I discuss this at length in my book-- but something missing does not mean something wrong.

I do not share the common “block universe” eternalist view of time either. What I argue in the book is that the presentist versus eternalist alternative is a fake alternative. The universe is neither evolving in a single time, nor static without change. Temporality is just more complex than either of these naïve alternatives.

TH: You argue that “the world is made of events, not things” in part II of your book. Alfred North Whitehead also made events a fundamental feature of his ontology, and I’m partial to his “process philosophy.” If events—happenings in time—are the fundamental “atoms” of spacetime (as Whitehead argues), shouldn’t this accentuate the importance of the passage of time in our ontology, rather than downgrade it as you seem to otherwise suggest?

CR: “Time” is a stratified notion. The existence of change, by itself, does not imply that there is a unique global time in the universe. Happenings reveal change, and change is ubiquitous, but nothing states that this change should be organized along the single universal uniform flow that we commonly call time. The question of the nature of time cannot be reduced to a simple “time is real”, “time is not real.” It is the effort of understanding the many different layers giving rise to the complex phenomenon that we call the passage of time.

294 comments:

  1. @JimV

    I am sorry if I sounded annoyed, it is just that I think other people other better equipped to explain this, both because I am not knowledgeable enough and because English is not my primary language. People with much better skills than me have written really good books about the topic, so I think you would gain more insight by reading literature than by asking me.

    With that out of the way: another attempt then.

    There are two arrows of time: the entropic arrow of time, and the psychological arrow of time, as we experience it in our everyday lives. The key insight is that if you think carefully about it, those two arrows must necessarily be the same. Step by step:

    - the psychological arrow of time is determined by memory
    - memory in any form requires that you are able to perform some sort of write and read operations
    - Write and read operations, regardless of the specific mechanism, requires 'energy'
    - But saying that something requires 'energy' is sloppy language: what we really mean when we say that, is that it requires a well of (energy with) low entropy. A battery is a good example of such a well.
    - Because of this, remembering anything is only possible if there exists such a well. What is more, the time direction that you remember as 'the past' is automatically aligned with the time direction into which the entropy flows away.
    - The reason we experience a flow of time at all is that entropy was extremely low at the time of the big bang. If the big bang would have happened in the future, we would remember the future. If there had not been a big bang or other low entropy origin, the universe would be a chaotic mess of radiation and possibly particles in thermal equilibrium, and remembering or doing anything would be impossible.

    Reading back your reply, I think the crux lies here:
    It is the fact that new events statistically add chances for entropy to increase that makes the arrow of entropy align with the arrow of time
    When you say this, think carefully about what you mean with that second "the arrow of time". Do you mean the entropic arrow, the psychological arrow, or something else?

    ReplyDelete
  2. weristdas said: The effect of gravitational time dilation is expected in about ten time larger effect than the effect of "kinetic time dilation".

    Not true. For symmetrical trips, the gravitational effect minus the “squared” part of the kinetic effect give about 118 nsec, and the directionally-dependent part of the kinetic effect adds (westward) or subtracts (eastward) about 157 nsec, which gives the prediction of +275 nsec westward and -40 nsec eastward. Both trips flew at the same altitude, so the difference between eastward and westward is entirely due to the kinetic (special relativity) time dilation. The gravitational effect only relates to the comparison with the ground clock.

    I would prefer if this probable bogus experiment is wiped out from the collective memory as allegedly strong indicator for time dilation, especially in respect of kinetic time dilation.

    It wasn’t a bogus experiment. You may have been misled by some fallacious criticisms that circulate on the web, such as the ones you cited in your earlier post. The recognizable jumps in the drift rates, which were characteristic for that kind of clock, caused the unadjusted data to only be in qualitative agreement with the prediction, but after correcting for the jumps using cross-clock comparisons, the results came into quite good agreement. In any case, this is of only historical interest, since today with modern clocks it’s possible to detect the motion effect of 20 mph, and the gravitational effect of half a meter of elevation. (Needless to say, these amusing demonstrations, as impressive as they are, don’t form the empirical basis of special relativity, which was actually conclusively established over a hundred years ago.)

    Note: "hafele keating relativity" returns nearly 30,000 hits e.g. with bing.

    Sure, it’s a popular topic, because non-scientists can picture an actual clock strapped into an actual airline seat and flying around the world. It was sort of a cross between a serious scientific experiment and a fun popular demonstration. Special relativity (Lorentz invariance) was thoroughly established decades before 1972.

    You should easily see that any different path of a slow motion (without passing different gravitational potentials) is identic concerning time dilation.

    Not identical. Any difference in path (through spacetime) implies difference in motion. Slow clock transport does not contradict this, because it refers to the limit as the speed approaches zero, but for any non-zero speed there is non-zero difference in motion. Of course, we can arrange for symmetrical deviations from inertial motion, so two clocks can read the same when re-united, but if they read differently they must have followed different paths through spacetime.

    I think you had made some simplifications.

    No, the elapsed time along any path is exactly the integral of sqrt(g_mn dx^m dx^n) along the path.

    ReplyDelete
  3. Erratum/addendum: I should have said that if there is a causal relationship, it seems more likely to me that the arrow of time causes the arrow of entropy than vice-versa. Mainly because it is statistically possible for entropy to decrease, but no evidence that I know of that the direction of time can reverse, i.e., that happenings can unhappen (as in that they never happened, not as part of a cycle). Of course, if they did they would leave no evidence, but it seems simpler to assume they don't.

    Entropy can decrease either by deleting previous happenings which caused to it increase or by new happenings which are statistically very unlikely, e.g., all the gas in container concentrating into a smaller volume by random kinetic actions. The first cannot be observed, the second will not alter the direction of time. Thus I see no evidence that entropy is what causes the flow of time (happenings). It should be strongly correlated with a past-to-future flow, but that just seems to be statistical theory in operation.

    ReplyDelete
  4. @sean s: "Time without change is stasis; change without time is absurd."

    Very much agreed. All the attempts to describe "timeless" abstract change have built in assumptions of time. First one thing is true, then another.

    Sorry for bringing up the dreaded philosophy again, but Kant's point here was that time is so fundamental to our experience that it structures how we think. Our thought process itself, is linear in time. A mind completely isolated from any physical reality would still be able to discover time.

    @JimV: "What I don't see as necessary is that entropy is forcing that order."

    Very much agreed. As I said in my very first comment on this thread: To me, thermodynamics arises as a consequence of particle behavior in time, not the other way around.

    I especially like your argument about how entropy can naturally go backwards. Time, as far as we know, not so much.

    I've been following the thread with great interest (even wrote my own blog post about it), and from where I sit, there hasn't been any convincing argument in favor of time being emergent or composite.

    One consideration for me: If we agree the Big Bang was an event that occurred, then surely there was a context, a set of physical laws, in which it occurred. I find no coherent argument that within this context, there was nothing that passes for time and space (call it meta-time and meta-space).

    If the BB occurred it necessarily occurred somewhere and somewhen. Perhaps for very meta definitions of where and when. And to me, this is a very compelling argument that time and space are fundamental aspects of reality. They came first.

    ReplyDelete
  5. Lorraine Ford;

    I see the universe as a system. If you attempt a systems analysis of the universe, it is clear that ... ” etc.

    Your view of the universe is your own, and it is not compelling. I don’t think you’ll find it helpful in your quest to understand the universe. I doubt the universe is a “system” in the way you perceive it.

    Good luck with your model, but I see no value in it.

    The numbers keep changing because the non-numeric things the numbers represent keep changing.

    sean s.

    ReplyDelete
  6. Tam Hunt;

    ... assessing the direction of entropy requires measurements in time. So how could measurements of phenomena in time be the physical/causal mechanism of time? This is another tautology ... Any attempt to craft some subsidiary process as a causal mechanism for time ends up in the same problem of trying to explain the emergence of time in time, yet another tautology.

    Exactly. Attributing time to change or entropy (or any other phenomena) are incoherent ideas. Time is necessary for either; so neither can be the “source” of time.

    sean s.

    ReplyDelete
  7. bud rap;

    In science, a lack of evidence for the physically meaningful existence of a thing that is claimed to have such existence is always significant.

    This is true only when we have some idea what evidence there should be. And to be clear: there is plenty of evidence that time is a fundamental existent. We experience time and cannot construct coherent explanations for how other phenomena could produce the effect: all such phenomena require time to occur in the first place.

    ... by stating that since the matter lies beyond the realm of the physical, it therefore lies beyond the realm of science.

    Time does not “lie beyond the realm of the physical”; physical things are universally subject to time. Time is as real as gravity.

    Such beliefs, however, have no claim on science because, as we agreed, science is based on empiricism.

    No, science is based on observation, empiricism, and/or logic. If no experiment is known, science does not end.

    The only thing I agreed to was that empiricism is not the only basis of science. I did not in any way mean to imply that it is an optional basis. No empiricism, no science.

    So an astronomer making observations is not engaged in science?
    A biologist recording population data is not engaged in science?
    Einstein, when using logic and imagination to create his theories was not engaged in science?

    You have a very limiting, constrained idea of what science is. The rest of us are free to ignore your restraints.

    But what then are the other bases of science? There is only one, logic (broadly inclusive of math and other forms thereof).

    Logic leads to the conclusion that time is fundamental. All other options are incoherent or absurd.

    ... we do not experience time in the exact same way we do not experience temperature. Both are simply human concepts that allude to the underlying physical phenomena we do experience.

    We do experience time. And without time the events and changes we experience cannot happen.

    The arguments you present for the necessary existence of time are, again, circular. They make no sense as a logical argument because they assume that time exists, as something other than a human concept, to begin with.

    Without time, change is not possible.
    There’s no logical way to make time derivative of anything else.
    There’s nothing circular about this.

    Can you give one example of change that occurs without time?

    You insist that time must exist otherwise change wouldn't happen. But it is a demonstrable fact that change happens. And you admit that there is no scientific, empirical evidence that demonstrates the existence of time.

    The fact that change happens proves time must exist. There’s no empirical evidence of what time is; but lots of evidence that time is and cannot be derived from other phenomena.

    You have said in the past that that all the evidence indicates that time is derivative; cite something please.

    sean s.

    ReplyDelete
  8. @Amos, responding to your 2:30 Sept. 16 post.

    Part 1

    the Lorentzian and Einsteinian interpretations do not differ empirically, because all the equations for making empirical predictions are identical. (As explained before, contrary to your beliefs, time dilation is an empirical fact, and neither the CMBR nor quantum entanglement imply any violation of special relativity.)

    Again, you're missing the key point that the difference between Einstein and Lorentz here is the physical interpretation of the mathematical formalisms. Lorentz suggested it was interaction with a preferred frame/ether that led to relativistic length contract. Einstein/Minkowski says it's a result of interaction with spacetime structure, another way of saying it's simply a consequence of the invariant c postulate. So, yes, if we confine ourselves to the Lorentz transformations and their predictions for any given real world situation, we can't empirically distinguish these two approaches. But when we think a little deeper we see readily that since the two approaches are adopting different versions of physical reality we can then test those different physical versions. If we can show evidence of a preferred frame/ether as a better explanation of observed relativistic effects we then may distinguish Lorentz and Einstein. That's what my draft paper does (link provided more than once above).

    As for time dilation, you're now just repeating mainstream dogma rather than actually examining my statements or answering my questions. That's fine, but I urge you look a little more deeply. I've provided a somewhat detailed analysis of three key papers purporting to find evidence of time dilation, in my draft paper. Rather than simply repeating assertions let me know if you want to delve deeper and discuss the substance of my arguments. As stated above, my examination of a number of key time dilation experiments reveals that each of the papers examined has serious and different flaws. I'm open, of course, to being shown the error of my ways on these papers, but no one has done so yet.

    As an example for you to chew on, check out Kelly 2000 and its re-examination of the Hafele-Keating unmassaged data, finding that the experiment did NOT in fact find evidence of time dilation at all. I discuss this experiment and the Kelly paper in my paper. https://www.researchgate.net/publication/239010566_Hafele_and_Keating_Tests_Did_They_Prove_Anything.

    CMBR and quantum entanglement do indeed have strong implications contrary to Special Relativity. Neither are a slam dunk falsification, but they add evidence against Einstein's interpretation. Nonlocality is defined as being faster than light. Quantum entanglement is defined in the Copenhagen Interpretation and Bohmian (and some others) as being nonlocal. Ergo: on its face quantum entanglement is a falsification of SR's prohibition against faster than light interactions, as Bohm himself stated and many others have recognized.

    ReplyDelete
  9. Doctor Boktor, thanks for your patience. This will just be to wrap up, then I doubt I will make any more comments on this thread.

    I don't disagree with any of your statements, but I see us as discussing different things. Low entropy as a starting point is necessary for there to be an interesting universe (which contains interesting things such as yourself)--agreed. Without that, nobody would be around to discuss time, so in that sense, time would not exist (as a concept considered by sentient biological beings). In the more general sense that there would be no happenings--there I must disagree (without being certain that is what you think; you may just be avoiding this question as not being significant). In my understanding, there will continue to be possible happenings after our universe is in heat death.

    According to what I postulated earlier, time is a counting of happenings. If there is no way to count, then it becomes meaningless to consider time--maybe that is what you mean. Anyway, it is not important that I understand you, although I would like to.

    ReplyDelete
  10. Chris Sonnack,

    I think we agree. Can you email a link to your blog to me?

    sean s.

    ReplyDelete
  11. @sean s: Heh, the world's most unread blog, but done.

    ReplyDelete
  12. @JimV

    " Anyway, it is not important that I understand you, although I would like to."
    What I meant to say is the following.

    Consider a universe in thermal equilibrium. Yes, there would still be time, there would still be changes going, things would still continue to happen.

    But: there would no longer be any process going that depends on the direction of time. Suppose that someone would tape a movie in such a universe. Impossible by definition, but consider it as a thought expirement.

    Now play the film backwards. It would be some sort of white noise, that looks exactly the same whether you play it backwards or forwards. Some facts would clearly manifest themselves:

    - the fact that the laws of Classical Mechanics remain exactly the same when you let time run backwards
    - the fact this holds true for relativity, electromagnetism, relativity and QM as well

    All* fundamental laws of nature remain exactly the same if you reverse time. It is impossible to distinguish past and future without a state of low entropy.

    Laws of nature related to friction, heat exchange and so forth are not fundamental, because they are statistical in nature, they are emergent behaviour of large groups of particles that only apply in the presence of low entropy.

    So without increasing entropy, yes there would still be time, but it would not have any meaningful sort of direction. Cause and effect would still be there in an abstract sense, but they would apply equally in both directions; the past would determine the future equally much as the future determines the past.

    And to make a stronger statement, in my opinion this means that even if time is fundamental, its direction is not; the psychological difference between the ´past´ and ´future´ directions of time are local circumstance, just like the approximate flatness of the earth is a local circumstance. ;-)

    I hope this makes it more clear what I was trying to say.


    *again, I heard there is some obscure exception related to certain types of particle decay; but after heat death all particles that can decay will already have done so, so it doesn't really change the argument

    ReplyDelete
  13. Yes, if we confine ourselves to the Lorentz transformations and their predictions for any given real world situation, we can't empirically distinguish these two approaches… but since the two approaches are adopting different versions of physical reality we can then test those different physical versions.

    That’s self-contradictory. If they are empirically indistinguishable (which they are), it isn’t possible for a test to distinguish between them. That’s what ‘empirically indistinguishable’ means.

    As for time dilation… re-examination of the Hafele-Keating unmassaged data, finding that the experiment did NOT in fact find evidence of time dilation at all.

    See my replies to “weristdas” on this subject. The “re-examination” you mention is fallacious, and the subject is of only historical interest, because modern repetitions of the airplane stunt, with much more accurate and stable clocks, have confirmed very precisely the relativistic prediction of time dilation - and this prediction applies to both the Lorentzian and Einsteinian interpretations. So, your beliefs are self-contradictory and empirically falsified.

    Please note that recent measurements are so precise they can detect the time dilation due to motion of just 20 mph and elevation of just half a meter. And even these demonstrations, though technologically impressive, are not particularly significant, since the empirical foundation of special relativity was fully established over a hundred years ago.

    CMBR and quantum entanglement do indeed have strong implications contrary to Special Relativity.

    No, special relativity just amounts to local Lorentz invariance, which is perfectly consistent with the global structure of spacetime and the CMBR; likewise quantum entanglement, far from contradicting the principles of special relativity, actually confirms them, since entanglement cannot result in any superluminal conveyance of energy or signaling. Indeed, quantum field theory was founded on the requirement to make quantum theory Lorentz invariant. Claiming that quantum field theory violates Lorentz invariance is nonsense. Spacelike-separated observables strictly commute, and local Lorentz invariance is strictly observed. It is well known that the non-classical correlations between entangled systems do not entail any superluminal action. In order to overthrow local Lorentz invariance you would need to cite some violation of local Lorentz invariance… but you can’t, because there is none.

    ReplyDelete
  14. @sean s

    Circular reasoning is a logical fallacy in which the reasoner begins with what they are trying to end with. - Wikipedia

    [ss] Without time, change is not possible.

    As an argument for the existence of time, that is a textbook example of circular reasoning. Since all of your logical arguments for time are of this form, and since you admit there is no empirical evidence (even imaginable), for time's existence as anything other than a human relational concept, you are left with nothing but a vivid emotional attachment to an existential belief that lacks any scientific meaning.

    That you find this existential belief satisfying in some way is perfectly alright with me - as long as you don't try to pass it off as science.

    ReplyDelete
  15. @Amos, no it's not self-contradictory. As I wrote very clearly and you quoted: "If we confine ourselves to the Lorentz transformations, they are empirically indistinguishable." If we instead look at the different versions of physical reality suggested by the Lorentz vs. Einstein interpretations we can indeed find many ways to distinguish these approaches (some of which tests have been done). I think I'll leave it at this on this point b/c you're simply repeating yourself now and not addressing my points.

    As for testing time dilation in the lab, I've pointed to problems with specific papers on this and you again repeat your assertions without addressing my specific comments. The lab tests of time dilation don't actually test time dilation, they assume its reality, and then discuss ways in which such lab measurements could be used to probe variations in the Earth's gravity field. This is a very different thing than actually testing, within the required accuracy, time dilation.

    As for quantum field theory not contradicting SR, you've got your causal chain backwards. QFT was developed specifically in order to reconcile SR and QM, so of course it fits with SR -- it was designed to do that. QM and quantum entanglement, outside of the specific mathematical framework of QFT, are in conflict with SR's prohibition against FTL influences. There is a growing cottage industry looking at ways to reconcile this apparent FTL influence with SR and I address these issues in detail in my paper.

    Let me ask you: does it make sense to you that there is some influence that apparently goes FTL, but that no "energy or signaling" is traveling FTL? If there's no energy or signaling what exactly is traveling FTL? How do you explain the FTL "correlations"?

    ReplyDelete
  16. @doctor boktor, you write: "All* fundamental laws of nature remain exactly the same if you reverse time. It is impossible to distinguish past and future without a state of low entropy." You're mistaking the fact that our laws or nature are written a certain way, which don't reflect empirical reality, as indicative of how reality actually is. Even the most simple of physical processes are irreversible. Prigogine's book (he won a Nobel Prize for this work), The End of Certainty, is all about this.

    Let’s consider a basic thought experiment: allowing diffusion to send an isolated molecule of hydrogen (H2) from one sealed glass container through to a connected (through a glass bottleneck) identical glass container, and then back again.

    Under traditional notions of the reversibility of such processes, the single molecule of hydrogen would be considered unchanged after this two-step movement to the second container and back to the first, reflecting a fully reversible physical process. However, this is the case only if we confine our examination and measurement to the spatial location properties of the molecule in isolation.

    More importantly, and the key point in this argument, is that while we may consider the hydrogen molecule in the connected sealed glass containers in isolation from the rest of the universe, the molecule and sealed glass containers are not in actuality isolated from the universe. They are very much connected to the rest of the universe through gravity and electromagnetic forces, which are long-range forces. While we can in such cases, for practical purposes, ignore such forces as negligible to the main inquiry, such forces are not in actuality negligible.

    The connected {hydrogen molecule + container + universe} is the system we must consider if we are examining reversibility in principle. And even if we accept arguendo that the hydrogen molecule and sealed containers are fundamentally unchanged by the two-step process described, the complete system {hydrogen molecule + container + universe} has of course changed in countless ways that are not reversible even in principle.

    As we scale up from this basic thought experiment to more complex situations, the interconnectedness of the universe reveals a fundamental irreversibility of practically all physical processes.

    See Prigogine for far more detail on this.

    The bottomline is that arguing, as you and Rovelli have done, that the reversibility of many of our current physical laws requires the universe itself to be reversible is to get it backwards. The laws need re-writing to reflect empirical reality, not the other way around.

    ReplyDelete
  17. bud rap,

    [ss] Without time, change is not possible.

    As an argument for the existence of time, that is a textbook example of circular reasoning.


    You think this because you still don’t understand the argument.

    Premise: All change occurs within time.
    Premise: No change is ever observed to produce time.
    Therefore: Change requires time.

    Conclusion: Without time, change is not possible.

    You might not agree with that, and it might even be flawed, but it’s not circular.

    ... since ... there is no empirical evidence, for time's existence as anything other than a human relational concept ...

    It’s not clear what your phrase “human relational concept” is supposed to mean here. Are you now claiming time requires human minds to exist? And your empirical evidence is ... what?

    It comes as no surprise that your last comment did not provide examples of empirical evidence showing time being derived from other phenomena; and that you did not provide any examples of “change without time”. How could you? Those are absurd concepts; yet your claims about time make such absurdities necessary.

    I do understand your desire to see direct, empirical data re. time; that would be the “gold standard” would it not? I’d love to have some too.

    But in the absence of that data, and our utter inability to even propose an experiment that might produce that “gold standard” data, rational, science-minded folks don’t just decide that time is derivative, emergent, composite, or caused by other phenomena. Where is the “gold standard” evidence for any of that? Nowhere. We all experience time; we never experience time being “made”.

    In the complete absence of direct empirical data about the true nature of time, all that is left is observation from experience, and logic

    In spite of all the energy you put into it, your claims about time fly in the face of both experience and logic. No change can happen unless time already exists; no empirically testable mechanism can be described that would “cause” time. How could it? Cause cannot precede effect unless time already exists.

    You talk at great length about the importance of empiricism to science (and generally your points are correct) but you seem to overlook that empiricism cannot evaluate “causes of time” unless time already exists.

    And you have yet to provide any valid, counter examples. Because none exist?

    Claiming time is created by change is like claiming gravity is created by things falling down.

    sean s.

    ReplyDelete
  18. doktor Boktor,

    Re “the laws of Classical Mechanics remain exactly the same when you let time run backwards - the fact this holds true for relativity, electromagnetism, relativity and QM as well” (4:02 PM, September 19, 2018):

    Not true: individual quantum events cannot be “run backwards”. And energy is not necessarily conserved in individual quantum events. Quantum events are unlike “Classical Mechanics… relativity, electromagnetism”.

    ReplyDelete
  19. On whether QM is local or contains an inexplicable FTL component, that was debated extensively in the comments to the post about Tim Maudlin's paper on the Black Hole Paradox. As chaff among the wheat there, I ventured to compare quantum entanglement as a physical property to color, as in, suppose Bob and Alice (who are far apart in space) are each sent a marble, known to have been selected randomly from a set of one red and one blue marble. Bob opens his package, sees a blue marble, and instantaneously knows Alice has received or will receive a red marble. Is this an FTL transfer? (For the response, see that thread.)

    Other than that issue, I think all current arguments have already been responded to on this thread, some of them many times. Most of them come down to whether one does or does not accept Special Relativity as confirmed theory. (E.g., according to SR, the direction of time - as in the order of events - is local, and there can be change without time if one is a photon.) (For that matter, two or more changes could happen instantaneously even if one is not a photon.) In any case, if people do not understand each other's logic or lack thereof, there is little point in arguing.

    (Do I deserve another fine for this comment? Probably, better pay it just in case. Also, no more comments by me for at least two days.) (Fine plus suspension.) (Before that, thanks again to Doctor Boktor.)

    ReplyDelete
  20. I see a lot of comments here about “running time backwards”. Although they’re interesting, I hope everyone realizes that they are only thought experiments (Gedankenexperiment? I’m not sure what the plural in German would be.)

    In reality, there’s no evidence that time ever actually runs backwards so the results of these particular thought experiments might be inapplicable in the real world.

    We might imagine an oak shrinking down and turning back into an acorn; but since we’ve never actually seen such a thing, the idea approximates a fantasy.

    Thought experiments are a valid exercise, but unless they lead to tangible results, they are liable to be sterile exercises.

    With regards to the nature of time, I doubt these thought experiments will prove useful. But do carry on.

    sean s.

    ReplyDelete
  21. Tam Hunt wrote: …the epsilon factor referred to above, which is assumed to be 1/2 in SR, indicating the same speed out and back. That's an untestable assumption…

    It isn’t an assumption, it’s a stipulation, representing the choice to use inertial coordinates, i.e., coordinates in which the equations of Newtonian mechanics hold good in the low speed limit. Naturally we’re free to use a different value of epsilon, but then we aren’t using inertial coordinates.

    There is no contradiction…. my point is that there is no actual time dilation.

    The contradiction is that you claim to believe in Lorentzian relativity and yet you deny the physical reality of time dilation, which is a prediction of Lorentzian relativity. And time dilation is an empirical fact.

    It's just a coordinate effect that disappears when we remove tautological assumptions or synchronization techniques.

    No, you’re failing to distinguish between passive and active transformations. Look, using Lorentzian relativity, work out the elapsed time on an ideal clock moving inertially from t=0, x=0 to t=10, x=5 and the elapsed time on an ideal clock moving inertially from t=10,x=5 to t=20,x=0 (all times in units of seconds and distances in light-seconds). How does this compare with the elapsed time for an ideal clock moving inertially from 0,0 to 20,0?

    …recent experiments that seems to find evidence of time dilation based on lab clocks at different heights (separated by as little as half a meter)… these experiments, which have been cited frequently as evidence of time dilation, actually just assumed time dilation and are looking instead to use time dilation to measure differences in terrestrial gravity fields.

    No, the experiments involved elevating the clock by a measured distance and noting the change in the rate of the clock in agreement with the prediction of general relativity. They also set the clock into motion at constant elevation and demonstrated the effect of the special relativistic time dilation due purely to motion.

    The same geophysicist acknowledged to me that these experiments can't even test time dilation in this type of experiment b/c the clocks weren't accurate enough until very recently.

    Surely you meant to say that the time dilation for such small differences in elevation (half a meter) or such low speeds (20 miles per hour) could never previously have been detected because sufficiently precise clocks were not available until recently. Again, Lorentzian and Einsteinian interpretations of special relativity are empirically equivalent, and the real physical effects have been abundantly demonstrated, so your beliefs are self-contradictory and empirically falsified.

    No it's not self-contradictory. If we look at the different versions of physical reality we can indeed find many ways to distinguish these approaches (some of which tests have been done).

    Either they are empirically indistinguishable or they are not. Remember, all your beliefs rest on your disproven claim that Lorentzian relativity predicts no physical time dilation, but it does, and this prediction is empirically confirmed. So your beliefs are self-contradictory and empirically falsified.

    The lab tests of time dilation don't actually test time dilation, they assume its reality, and then discuss ways in which such lab measurements could be used to probe variations in the Earth's gravity field.

    That’s nonsense. We move a clock at 20 mph and note that it is slow by the predicted amount. We elevate the clock by half a meter and note the predicted rate change. Your denial of time dilation is abundantly falsified.

    QM and quantum entanglement, outside of the specific mathematical framework of QFT…

    That’s nonsense. Non-relativistic quantum mechanics is falsified, so the only viable quantum mechanics today is relativistic quantum mechanics, and quantum entanglement is not “outside the mathematical framework of QFT”.

    ReplyDelete
  22. It is known that energy is not necessarily conserved in a quantum event, and that quantum events are unlike Classical Mechanics and relativity.

    I ask doktor Boktor and Amos:

    We have been discussing aspects of reality relevant to the issue of “time” and “change”.

    An individual quantum event cannot be “run backwards”. True or not true?

    ReplyDelete
  23. Lorraine,

    The best answer I can give you is that in QM, the standard model is said to be symmetric under reversal of charge, time and left/right directions of space. That is known as CPT symmetry and more subtle than time symmetry, but still similar.

    With 'event', I would guess you mean 'observation' and the collapse of a wave function? I have to be honest - I just don't know how this relates to CPT symmetry, that is something I would like to know as well. In any case, as I tried to say before, I am not an expert and I am not here to 'win' a yes or no discussion. It is hard enough to communicate over the internet in a productive way.

    I have never heard of violations of conservation of energy in QM, so if you can refer me to a source where I can read up about it, please do.

    JimV: thank you for your patience as well, it was nice talking to you

    ReplyDelete
  24. doktor Boktor,

    Surely any “quantum” situation where there is an unpredictable jump to an outcome seemingly selected from a range of possible outcomes, is a situation where the outcome numbers for the variables are ratcheted in one direction only. (This is assuming there are no hidden variables determining this outcome jump.) Note that one seemingly cannot assume that 1) outcome possibilities really do exist; 2) the possibilities would be the same if you wanted to reverse the situation (i.e. the situation can’t be symmetrical); and 3) the unpredictable jump to an outcome seemingly selected from a range of possible outcomes would reverse the outcome. No matter what the higher-level statistical equations describing a system might say, at a more fundamental level the outcome numbers have one direction only. (And seemingly the time variable is derived from these changing numbers.)

    Re conservation of energy: I guess I’m merely saying that energy conservation may be temporarily violated (this fact about reality is mathematically represented in the uncertainty principle); but also that systems are not isolated, so that this violation may have ramifications in a larger system; this is a one-way fundamental-level process where numbers are changed. (This paper might be relevant: Extending Noether's theorem by quantifying the asymmetry of quantum states, Iman Marvian & Robert W. Spekkens, https://arxiv.org/abs/1404.3236v1 )

    ReplyDelete
  25. Lorraine,

    For what little my opinions are worth, I agree that, for example, virtual particles in QM are said to violate energy conservation but for only a brief time, consistent with the Uncertainty Principle. Quantum events being uni-directional in time seems a bit paradoxical since "classical" (macroscopic) events are just combinations of a lot of quantum interactions and classical events are said to have the possibility of occurring with time-reversibility. However, maybe your point is that that the classical assumption is wrong.

    As I have said too many times, I think most of those who developed the classical laws, such as Newton, implicitly assumed that they were uni-directional since they assumed time is uni-directional; and in practice, those of us who calculate models using those laws always use the forward form of the time derivatives to go from times we know to those we do not. Of course, we all could equally well assume time values were counting down, like a rocket launch, instead of counting up, and for that reason the laws have to seem bi-directional in time to account for that possible coordinate transformation. So I tend to think all our laws will need to have that form, for that reason (the universe should not care whether we count time up or down), but perhaps my supposed implicit assumption will be made more prominent in those laws. E.g., use the forward form of the time derivatives if you count up, and the backward form if you count time down.

    I was thinking earlier that General Relativity does not always allow time-reversability (in the sense of events occurring in reverse order, not just the above coordinate transformation) since as I understand it, once two neutron stars approach, fall into a decaying orbit, and combine into a black hole, they cannot escape the black hole to do the events in reverse order. However, after our host combines GR with QM in a better model of quantum gravity, maybe they could (with a vanishingly small probability). So I decided that wasn't a good counter-example to use.

    ReplyDelete
  26. JimV,

    My point is not “that the classical assumption is wrong”, but that one has to look more closely at a more fundamental level of reality to see what is actually happening when it comes to the “direction” of time. As you say, the laws are “bi-directional”, and the “laws will need to have that form”: i.e. the laws themselves cannot generate a direction for time. I am contending that the numbers have to be looked at separately to the laws; and that the numbers are the source of the direction of time.

    There seems to be an assumption something like this: law of nature equations and initial value numbers are the only 2 factors you need to generate all future numbers for all variables. But the example of a computer system gives the lie to the notion that the numbers take care of themselves: the stream of numbers that are output by a computer requires both an algorithm to control the equations and generate the numbers, and electricity to power the computer.

    So, number change does not just “happen”; the law of nature relationships cannot generate number change or a direction for number change; the law of nature relationships can only generate the consequence of a separately occurring number change. I am contending that individual quantum events continually generate the required “uni-directional” stream of numbers for variables; and that the law of nature relationships generate the consequence of these changing numbers.

    ReplyDelete
  27. Second reply to Lorraine Ford on quantum event time-reversibility:

    I have thought of an example which I think shows that quantum events can happen in reverse orders. This is probably not what you are looking for, but personally, I think that is the only sense in which anyone is claiming that laws of nature are time-reversible. That is, Doctor Boktor, following Sean Carroll, seemed at first to me to be claiming that only entropy stops time from going backward as well as forward, but after our discussion I think they mean the practical likelihood of seeing certain reverse-orders (broken eggs recombining) not that time really reverses direction during such unlikely but possible events.

    So for example the laws of Quantum Electrodynamics say that electron A can emit a photon which is absorbed by electron B, and also that the reverse can happen, and electron B can emit a photon which is absorbed by electron A. (That's my layperson's example.)

    Your "time is caused by numbers changing" seems identical to me to my "time as we know it (and all we can know it as) is the relation between counts of happenings, e.g., the time I spent working for GE could be measured in orbits around the sun (years)". If I am correct in this interpretation, then we agree that events/happenings/changes are the fundamental basis of what we call time. Which (to have that going for me) would be nice. (As Bill Murray might say.)

    ReplyDelete
  28. JimV,

    I agree that they can only mean “the practical likelihood of seeing certain reverse-orders (broken eggs recombining) not that time really reverses direction”. In this scenario, there seems to be no genuine impediment to eggs recombining except likelihood.

    Re “the laws of Quantum Electrodynamics say that electron A can emit a photon which is absorbed by electron B, and also that the reverse can happen, and electron B can emit a photon which is absorbed by electron A”:

    I think that a quantum event should be looked at this way: For at least some of the variables representing a quantum event, the outcome numbers can’t be predicted: there is merely a probability distribution on the set of possible outcome numbers. What outcome number is actually “chosen” is inherently unpredictable. So even though it might appear on the surface that an outcome can be reversed, in the quantum event scenario there is a genuine impediment to events being reversed because of the unpredictable selection from possibilities aspect: at least some of the numbers for some of the variables will be different. So I’d say that an individual quantum event is a fundamental-level uni-directional event, which outputs new and unpredictable numbers. And when you look at the universe as a system (somewhat analogous to a computer system), there is nothing else causing number change except quantum events

    I didn’t say that “time is caused by numbers changing”: the abstract fact of number change can’t cause anything. I think that time is higher-level information because time is seemingly derived from knowledge of regular change (so I seem to agree with you about that). Time is a variable with a unit, and seemingly all fundamental-level variables must have some sort of delimiter or unit. Maybe the only natural, non-arbitrary delimiter is a number jump due to a quantum event.

    ReplyDelete
  29. Amos 6:21 PM, September 17, 2018,

    "Both trips flew at the same altitude, so the difference between eastward and westward is entirely due to the kinetic (special relativity) time dilation."

    We talk about the H&K-Experiment? This experiment was conducted with commercial flights, with several starts and landings, with sometimes changes of the planes, ...

    There were no "both trips", there had been multiple trips which had been sticked together like broken vases or so.

    It is not reasonable to handle the H&K experiment as unimpeachable, like you try to suggest.

    (Until now) we can't be sure about kinetic time dilation, same as we can't be sure about kinetic length dilation. The rest is pure wishful thinking.

    ReplyDelete
  30. We talk about the [Hafele-Keating] Experiment? This experiment was conducted with commercial flights, with several starts and landings, with sometimes changes of the planes, ...

    Yes, and even during the flights they were rarely moving directly eastward or westward, but at various angles. All of this was taken into account in determining the predicted time dilation.

    There were no "both trips", there had been multiple trips which had been sticked together like broken vases or so.

    The phrase “both trips” refers to the eastward trip around the word and the westward trip around the world. It’s understood that each trip made stops along the way. Again, both trips spent roughly the same amount of time at various altitudes, so the difference between the eastward and westward clocks was almost entirely due to the kinetic (special relativity) time dilation. The eastward clock was traveling – on average – faster than the westward clock, due to the Earth’s rotation.

    You stated that the gravitational effect was “ten times greater” than the motion effect, but that’s completely wrong, because nearly the entire difference in elapsed times between the two traveling clocks was due to their different states of motion, not to different elevations. (The difference with the ground clock included significant effect due to elevation.)

    It is not reasonable to handle the H&K experiment as unimpeachable, like you try to suggest.

    Their own paper described the drift rates of the clocks, and the ocasional abrupt shifts in the drift rates (which were characteristic of those clocks), and how they detected and corrected for them. You haven’t provided any rational objection to their analysis. Also, you seem to ignore the more recent repetitions with much more accurate and stable clocks, confirming the result, so the original HK result is of only historical interest.

    (Until now) we can't be sure about kinetic time dilation, same as we can't be sure about kinetic length dilation. The rest is pure wishful thinking.

    Our knowledge of relativistic time dilation doesn’t rest on Hafele’s fun little airplane demonstration, nor even on the more recent repetitions with more precise clocks. The empirical foundation of special relativity (local Lorentz invariance) was conclusively established over a hundred years ago. This entails time dilation, length contraction, the inertia of energy, and all the other relativistic effects. You can read about the empirical foundations of special relativity in any good book on relativity. If you are interested only in macroscopic clocks, today there are clocks accurate enough to detect the time dilation due to motion of just 20 mph.

    ReplyDelete
  31. Amos 12:22 PM, October 01, 2018,

    first you claim that there was just an easy and clear experiment with clear results. After my protest you go deeper but again you suggest that there are no problems with the H&K experiment. You admit that there had been several starts and landings, travellings not exact to east or west direction, different altitudes as commercial flights normally shows, etc. But again your statement is nothing more than: "Trust me! The experimenter would have done their job well, yet.".

    No, I dont't trust baseless statements. The composing of the different pieces of the experiment have to lead to a much greater error than they reported, because they have to summed up.

    ReplyDelete
  32. I'm curious about all these questions about the "H&K Experiment"; is it the only experiment physically testing time dialation? Do doubts about its methodology invalidate its results or only limit its accuracy?

    sean s.

    ReplyDelete
  33. Late to the discussion here, but I just finished reading the book. The concept of 'emergence' seems to really be a sticking point in this interview. Emergence doesn't imply time.

    Chemistry 'emerges' from physics because the laws of chemistry stand on their own, while simultaneously having the laws of physics as an underpinning, but the reverse isn't true - physics doesn't require the laws of chemistry as an underpinning.

    That doesn't mean (a) that chemistry 'came after' physics in time, nor that (b) chemistry is 'less true' than physics. But it is separate from physics, because it contains truths that aren't relevant to physics while still being 100% true. Their truths exist in independent domains, but there is a direction from one domain to the other - that direction is 'emergence'. No time needed.

    Using this analogy, I think that when CR talks about 'emerging', he's saying that time acts the way a truth in chemistry acts- it doesn't have to be found in fundamental physics for it to be true, so we should get comfortable with the fact that we can't find it there. (CR explicitly says in the book- and in the interview- that he doesn't think time is an illusion, but the interviewer keeps throwing that accusation at him. I see why CR is getting a bit testy).

    The world is full of real phenomena for which fundamental physics has no explanatory power- that's not the same as saying they're 'illusions'.

    I thought CR's book did a fantastic job striking the balance. He's suggesting that we haven't got a properly rigourous definition of time- we use a single word for a set of partly-understood separate things. Because of this, we don't understand the domain(s) that time's truth exists in- fundamental physics contains partial foundations for time, just as it does for igneous rock and the US Congress. But it doesn't contain, or predict, time itself. We can allow that without throwing out empiricism.

    I see why people are afraid for empiricism in this 'post-truth' time, but we have to be careful about being knee-jerk about it. We're at a point in intellectual history where we're starting to understand the ways that truths can exist in domains. This is what 'emergence' is about. I think that this is actually the next big frontier in science, because it corrects for the ways that science and empiricism *appear* to have limits to their range of inquiry (when actually they don't), and might help bridge the apparent gap between 'science' and intuitive / common / utilitarian knowledge.

    ReplyDelete
  34. Rob Langford (thanks for your comment), I got a similar impression just from the interview (haven't read the book although it sounds interesting and maybe I will). An additional source of testiness emerged over the span of this long thread (which I don't recommend reading) though, which is that it appears that Tam Hunt does not accept Einstein's Special Relativity and its implications about the nature of experienced time. That makes him rather a biased reviewer of any main-stream book of science. I guess it might do us good to encounter contrary opinions every now and then, but that seems almost in the flat-earth category to me.

    ReplyDelete
  35. first you claim that there was just an easy and clear experiment with clear results.

    Not true. My first comment on Hafele-Keating in this thread was when someone said their data had been “massaged”, and I replied “Of course the data was massaged, as the effect was just at the limits of capability at the time… but the results were legitimate, and later repetitions with much more precise clocks have confirmed the results beyond all doubt, and of course we now have GPS clocks and so on.”

    After my protest you go deeper but again you suggest that there are no problems with the H&K experiment.

    As mentioned before, the drift rates of the clocks were much greater than the magnitude of the relativistic effect, so some delicate analysis was required to extract the time dilation “signal” from the background drift. Their paper explains in detail the drift rates and the shifts in drift rates, and how they detected and corrected for these. They also acknowledge numerous sources of uncertainty that could contribute to the error bars. Again, if you want more precision, with less reliance on post-test data correction, look to the later repetitions with more modern clocks, having much better accuracy and stability. Modern clocks can detect the time dilation due to motion in a lab at speeds as low as 20 mph.

    You admit that there had been several starts and landings, travellings not exact to east or west direction, different altitudes as commercial flights normally shows, etc.

    It’s strange to call that an “admission”. Everyone knows they took commercial flights. Also, it’s not difficult to account precisely for the variations in altitude, speed, and direction. The difficulty was in accounting for the drift and rate jumps of the clocks.

    But again your statement is nothing more than: "Trust me! The experimenter would have done their job well, yet.".

    Not at all. The paper describes in detail how they accounted for all those effects. Of course, they could have falsified their data or cheated with their analysis, but that’s true for any experiment. That’s why we place more confidence in experiments that have been repeated by independent groups – as this one has been. And, again, these airplane stunts are just fun demonstrations. The real empirical basis of special relativity is all the multitude of tests and observations confirming local Lorentz invariance.

    The composing of the different pieces of the experiment have to lead to a much greater error than they reported, because they have to summed up.

    Again, it’s not difficult to precisely account for the variations in speed, altitude, and direction during the trips. The difficulty was in accounting for the variable drift rates of the clocks, and they described how they did this.

    ReplyDelete
  36. @Rob Langford, I address that kind of emergence in my questions with Carlo. The kind of emergence you and he referred to, an ostensibly non-temporal emergence, requires a conscious observer to notice the purported emergence. The laws of chemistry "emerge" from the laws of physics only in terms of an intellectual structure that attempts to identify the regularities of certain kinds of physical things. All of these aspects of this type of emergence require, of course, a conscious observer. Conscious observers are by definition in time. I hope that point is clear. Similarly with Carlo's alleged emergence of time from more basic physical laws that are wholly non-temporal: such emergence is simply an observational feature of our structure of physical laws. If, on the other hand, you or he would suggest that such emergence is a temporal kind of emergence, taking place in the physical world, then we are obviously back to the tautology of suggesting that time emerges in time. Ergo: we should look for other ways to explain the obvious passage of time in the human world and in the world all around us. That is what has prompted my own investigations into alternatives to Einsteinian relativity (such as the original Lorentzian interpretation or various neo-Lorentzian approaches.

    Carlo got a little testy with me b/c I suggested that his argument isn't logical. He argued that while time is an illusion at the fundamental level, it emerges somewhere before the human level. I suggested his notion(s) of emergence didn't make sense and hence the impasse.

    As for empiricism, I would suggest that modern physics has become strangely anti-empirical (and Sabine's book is a lengthy treatment of this trend), including when it comes to relativity theory. We need to return to a stricter kind of empiricism if we are to move ahead in answering the big questions.

    @JimV: it is more the other way around in terms of a religious-like belief in the veracity of Einsteinian relativity, which renders even reasoned, evidence-based debate about relativity somehow beyond the pale (akin to Flat Earthers as you state). You'll notice if you've read this thread that when I've engaged Amos (the person who I've chosen to engage with most b/c of his evident knowledge of these issues) on the sub-issues, I've raised specific objections about specific papers. He's resorted to dogma and generally accepted views each time, not bothering to address my specific points. The fact is that the evidence for an actual physical time dilation, rather than simply a coordinate effect akin to switching time zones, is very poor. The recent experiments in labs purporting to show time dilation on lab benches are like the Hafele-Keating experiment in that the limits of their experimental accuracy can't even show the alleged conclusions. But the latest experiments are different in a key way: they weren't even trying to establish evidence for time dilation, they simply assumed it. Rather, they were trying to show how time dilation could be used for geodetic surveys as a new tool. This would be fine if it were stated explicitly, but it hasn't been.

    More generally, if not agreeing with the mainstream views makes one a "biased reviewer" of a proponent of mainstream views, heaven help us in terms of having any reasoned debate and moving beyond groupthink...

    ReplyDelete
  37. I've raised specific objections about specific papers. He's resorted to dogma and generally accepted views each time, not bothering to address my specific points.

    Not true. Each of your “specific points” has been debunked here in detail, with no resort to dogma at all. Remember, you disputed the Hafele-Keating results, but their paper explains how they corrected for the clock drift, so your objections are unfounded. Also, you admitted to being unaware of the repetitions with even more accurate modern clocks, confirming their results. You earnestly asked me for references, promising to study them with an open mind. I provided the references and you had no further comment.

    Another of your “specific claims” is that measurements of the speed of light are circular, because you believe they rely on synchronization established by assuming light speed is c. I explained in detail why that is false. For example, you claim that slow clock transport is equivalent to light synchronization by definition, but that's untrue. Again, inertial coordinate systems are, by definition, systems in which all the laws of physics hold good in their usual homogeneous and isotropic form, and this fully determines the synchronization of the time coordinate based purely on isotropy of mechanical inertia – without any reference to light propagation. That’s the key insight you are missing.

    Now, the functioning of any clock is governed by the laws of physics, which take the same form in every system of inertial coordinates, so slow clock transport happens to be one of many methods of establishing the time of inertial coordinates. Once we have constructed such a system (with no reference to light propagation at all), we find empirically that the speed of light is c in terms of those coordinates. This is not tautological, it is a very significant empirical fact with enormous physical implications (e.g., E = mc^2). You can read about this in any good book on the foundations of special relativity.

    Another of your “specific claims” is that time dilation is not physically real, and that even the most modern atomics clocks, that claim to show the relativistic time dilation of 20 mph and half a meter of elevation, actually don’t show this at all. You base your belief on an interview with a geophysicist who told you that people might use such clocks to measure geological configurations. Needless to say, your “reasoning” is a complete non-sequitur. You disregard all the other evidence based on similarly specious “reasoning”.

    More fundamentally, all your beliefs rest on the misconception that Lorentzian relativity does not predict real physical time dilation. You keep repeating that “relativistic time dilation is just a coordinate effect, like changing time zones”, but that is completely wrong, ignoring the difference between passive and active transformations. To clarify this, I’ve asked you repeatedly to tell me what you think Lorentzian relativity predicts for the elapsed time on ideal clocks following two different paths between two events. You never answer.

    Again, according to your understanding of Lorentzian relativity, in terms of inertial coordinates t,x, in units of seconds and light-seconds, what is the elapsed time on an ideal clock that moves inertially from (t,x) = (0,0) to (10,5) and then from (10,5) to (20,0)? The answer, obviously, is 10sqrt(3) = 17.32 seconds, whereas a clock moving directly from (0,0) to (20,0) would show an elapsed time of 20 seconds. Thus Lorentzian relativity predicts different amounts of elapsed times for the two paths between two events. Do you dispute this?

    ReplyDelete
  38. I’ve fallen behind; too much to do at work!

    Rob Langford: “Chemistry 'emerges' from physics because the laws of chemistry stand on their own, while simultaneously having the laws of physics as an underpinning, but the reverse isn't true ...

    Agreed. The “emergence” of chemistry from physics is an example of an “emergence without change”; that’s why this kind of emergence can be seen as “timeless” because nothing changed but the observer’s POV.

    Tam Hunt commented on this too: “... an ostensibly non-temporal emergence requires a conscious observer to notice the purported emergence. The laws of chemistry ‘emerge’ from the laws of physics only in terms of an intellectual structure that attempts to identify the regularities of certain kinds of physical things.

    Rob Langford: “The world is full of real phenomena for which fundamental physics has no explanatory power- that's not the same as saying they're 'illusions'.

    Quite right.

    sean s.

    ReplyDelete
  39. @Amos, part 1. We've been over your points and I've made additional points that you then chose not to engage with (right when we were getting down to the nitty gritty). I'll send followups on each issue you've mentioned, with quotes and critiques of each paper.

    As for "all your beliefs rest on the misconception that Lorentizan relativity does not predict real physical time dilation" this also is inaccurate. I've made clear in my comments here and in my paper that I've linked to a number of times that my arguments for Lorentzian rel. don't rest on time dilation being a coordinate effect only. Rather, my arguments rest primarily on the various empirical issues that I raised with respect to a cosmic frame, cosmological principle falsification, conflict with QM, etc. The arguments about time dilation as a coordinate effect only are additional but not necessary for my case. If it turns out that I'm wrong and time dilation is a real physical effect that doesn't affect my other arguments in favor of Lorentz rel.

    In your example of clock time dilation, you'll get the same clock reading on both clocks (20 seconds elapsed) if there isn't any actual time dilation. What I've been arguing is that there is no real physical time dilation b/c the "time dilation" of separated clocks in different inertial frames (not separated and brought back together clocks as in your thought experiment) measured is simply an artifact/artifice of the transformations and coordinate systems we've chosen. Lorentz made that clear but Einstein went the opposite direction and argued that this "fiction" that Lorentz created was in fact how time really worked (physically). But it doesn't.

    The transformations allow translations between different inertial frames (with the gamma [Lorentz] factor in the transformations that relates c to v, with c as the top possible speed, necessarily making "time dilation" the result of v approaching c), but the "time dilation" that results is purely a function of an assumed invariant c. For Lorentz, c was actually invariant in the preferred frame only (the fixed ether), but for Einstein it was invariant in all inertial frames. And that is the sum of their differences that really count. Lorentz made this assumption in order to allow for a workable translation procedure between inertial frames, for convenience. Einstein took it as ontologically significant, when it's not.

    As I've mentioned, Galison's book Einstein's Clocks, Poincare's Maps, is quite good on this history.

    More generally, what matters is not a thought experiment about time dilation, as you've offered, but actual experiments trying to measure time dilation. So let's look at a few and maybe you can point out where I've gone wrong.


    ReplyDelete
  40. @Amos, part 2. #1 Hafele-Keating paper. As I've mentioned, they do not actually correct for the clock accuracy deficiencies. The HK experiment had many serious issues from the outset, as their 1972 paper itself describes. The authors identify two main experimental accuracy issues: 1) the fact that they were measuring effects on the order of 0.1 microseconds per day and their machinery's accuracy was only within 1 microsecond per day; 2) in correcting the data for this issue they needed to also correct for unpredictability in expected drift in each clock, which they attempted to do with two different methods discussed.

    With respect to the first method for correcting for naturally-occuring time drift in the four clocks employed for the experiment, "the average rate method," the authors state (p. 169): "Reliability of results with the average rate method, however, depends on the unlikely chance that only one rate change occurred during each trip and that it occurred at the midpoints. Furthermore, there is no obvious method for estimating the experimental error. Nevertheless, the average rate method does produce convincing qualitative results."

    The last sentence is rather incredible given the first two sentences.

    With respect to the second method, the authors state (p. 177): "An analysis of these data revealed the times and magnitudes for correlated rate changes during each trip. Thus significant rate changes were identified and ascribed to each clock. A piecewise extrapolation of the time trace for each clock relative to MEAN(USNO), with proper accounting for these identified rate changes, then produced the relativistic time differences [observed]."

    We have to dig a bit deeper to find why this method, rather than being an appropriate adjustment, seems instead to be a strong example of cherry picking the data. Kelly 2000 ("Hafele and Keating Tests: Did They Prove Anything?" Physics Essays 13.4 (2000): 616-21. Web.) looks at the original data collected by HK from the four cesium clocks used in the experiment (this data was not published in the original paper), after the author request the original report from the US Naval Observatory, and concludes (emphasis added):

    "The [US Naval Observatory] standard station had some years previously adopted a practice of replacing at intervals whichever clock was giving the worst performance. On a similar basis, the results of Clock 120 [one of the four used by HK] should have been disregarded. That erratic clock had contributed all of the alteration in time on the Eastward test and on the Westward test, as given in the 1971 report. Discounting this one totally unreliable clock, the results would have been within 5ns and 28ns of zero on the Eastward and Westward tests respectively. This is a result that could not be interpreted as proving any difference whatever between the two directions of flight."

    Accordingly, under Kelly 2000’s re-examination of the raw data, it seems that we should accord little to no weight to this now iconic experiment purporting to find strong evidence of physical time dilation – that is, real differences in the elapsed time of traveling clocks.

    ReplyDelete
  41. @Amos, part 3. You've mentioned a few times now that clock synchronization choices don't matter as long as we're only considering inertial frames. I tracked down this assertion to Will (1992) https://sci-hub.tw/10.1103/physrevd.45.403.

    I stated in my subsequent response, which you never further responded to:

    "And in terms of your suggestion that any kind of synchronization technique yields the same speed of light figures, I don't believe this is correct. Are you referring to Will 1992 "Clock Synchronization and Isotropy..."? If so, see p. 45 and its statement about the result being independent of the clock synchronization technique BUT also requiring an assumption about length contraction, so the assumption chosen again dictates the outcome, which is the same as my earlier point about being unable to measure 1-way or 2-way speed of light without using some convention/assumption about synchronization."

    It seems pretty obvious to me that of course synchronization choices matter. That was Einstein's entire method in his famous 1905 paper: using light-signal synchronization in order to define time and simultaneity and then assuming that the consequences of this method should be viewed as ontological rather than just coordinate effects.

    A common sense alternative for synchronizing distant clocks, which was in fact used for the famous longitude problem solution, is to simply synchronize them at the starting point and assume that the clocks are accurate enough to not deviate significantly as they're moved away from each other. Any assumption that "moving clocks run slow" or are affected by gravity is begging the question about relativistic effects and a no-no.

    I agree that Einstein synchronization makes good sense in many circumstances as a practical technique, and it arose as you probably know in the context of how to coordinate train schedules and clock towers using connecting wires to send EM signals for synchronization (again Galison's book is excellent on this history).

    But we shouldn't take such convenience as indicative of the ontology of time or space. It's just a convenient method but its assumption that the speed of light is the same in both directions is contrary to everything we know about the transmission of signals in any medium. So why would light be different?

    ReplyDelete
  42. @Amos, part 4. Let's look now at Chou et al 2010, (https://tf.boulder.nist.gov/general/pdf/2447.pdf) which is the paper we've discussed previously that claims to find gravitational time dilation with a change in height of only 33 centimeters in the lab setting, due to the differences in Earth's gravitational field.

    I explained that I'd discussed this paper at length with a CalTech geophysicist, David Stevenson (I should have said planetary scientist since that's his preferred designation). This wasn't some random guy, he's actually partnered with the authors of the Chou paper in creating the geophysical tools I mentioned. I had reached out to him because I found him online when I searched for an expert on Earth's gravitational fields. My intention was to ask him how much variation there is in that field and to query him about the reliability of experiments attempting to measure grav time dilation on Earth's surface with respect to a nonconstant grav field on Earth's surface, due to all of the obvious differences in the topography of the Earth that will lead to different values for the field at different locations. He responded that he was actually working with Chou et al on implementing the proposed new generation tools for measuring the Earth's grav field. He didn't think there were any methodological issues with Chou et al and he was clearly very excited about the prospect of developing these new tools.

    When I asked about experimental error and the possibility of variable grav fields being a confounding factor in the Chou experiments, he eventually stated to me the remark that I repeated here earlier in this long thread, which I found incredible: "The validity of gravitational time dilation is not in doubt and not the point. (Our use of GPS validates its validity all the time). Rather, one accepts Einstein and then uses the observed time dilation to make inferences about the gravity field." So even though Chou et al 2010 and similar papers seem to be papers testing the reality of grav time dilation, I learned from my dialogue with Stevenson that in fact the experiments published to date don't have the experimental accuracy to find the grav time dilation they claim to find. And that new clocks are only now being developed that do have such accuracy. But these experiments with the new clocks either haven't taken place yet or have taken place but not been published yet.

    Here's what Stevenson told me:

    "Where did you get the idea that the experiments we're discussing are designed to test gravitational time dilation as a new area of research? There are many uses of high accuracy atomic clocks, and there is basic physics (nothing to do with gravity) of interest. There is also the opportunity to use gravitational time dilation (not test it, we know it already). Perhaps this is semantics. But to me a "test" is where you are trying to find a failure in the theory. Whereas a "use" is where you accept the
    theory and apply it. The excerpt you sent is a use , not a test (that is, the authors accept the validity of general relativity). There are of course people who devote time to testing general relativity and it has proved to be a thankless task (the theory is spectacularly successful in multiple ways). One could use the new clocks to do a test but frankly this is a boring activity, or at least an activity that has very little prospect of a payoff. Devising a use is much more interesting."

    FYI I also reached out to Chou a couple of times but never got any response from him.

    ReplyDelete
  43. Groupthink can be right or wrong. Science attempts to find groupthoughts that are correct. When a huge majority (near 100%) of scientists accept a thought, that argues for it rather than against it. I don't know of a better standard for empirical questions.

    On a practical level, the assumption of a fixed speed of light in vacuum is being used hourly in calculations taking place all over the world, and no one is noticing any discrepancies from the results of those calculations (that aren't resolvable). For example, NASA has had several probes wandering the solar system, proceeding at known accelerations, and sending back radio signals. Those signals arrive at times consistent with a constant speed of light, despite various velocities of the probes. In engineering we have a saying, "if it ain't broke don't fix it." Special Relativity works remarkably well (as does General Relativity--whose gravity waves travel at the speed of light which produced the seven-second delay in arrival time between two LIGO sites). That is why Dr. Stevens and others aren't interested in debating it.

    When there is consistency in a lot of different calculation results with observation, a groupthink consensus emerges; just as when a lot of counts of different happenings (e.g., days versus years matching at a ratio of about 365.25) are consistent, a general concept of time emerges. None of these individual cases could not be explained by other means (including human error), but as long as a unifying assumption seems to work there is no reason to abandon it.

    The above seems so obvious that I should pay a fine for posting it, so I will.

    ReplyDelete
  44. @JimV, the point of the "groupthink" term is to highlight the more pernicious version of consensus--a tendency to shut down any debate on consensus items. Yes, of course, general consensus is how science proceeds, but equally obvious is the need to preserve the ability for reasoned critiques and debates to be heard. We have now been in an era where groupthink has often squelched such reasoned debates on a number of issues. I again refer you to Sabine's book for a longer treatment of these issues. And Smolin's book The Trouble With Physics. And then to round it out read Kuhn's The Structure of Scientific Revolutions for his distinctions between "normal science" (~95%) and "revolutionary science" (5%).

    ReplyDelete
  45. TH wrote: My arguments for Lorentzian relativity don't rest on time dilation being a coordinate effect only. Rather, my arguments rest primarily on the various empirical issues…

    But there are no empirical issues, because special relativity and neo-Lorentzian relativity are empirically indistinguishable. Your only purported counter-examples are

    …cosmic frame, cosmological principle falsification, conflict with QM, etc.

    But we already established that special relativity (i.e., local Lorentz invariance) does not conflict with any of those things. See the previous messages where your misconceptions about these topics were explained in detail (e.g., Lorentz invariance is local not global, quantum field theory is explicitly Lorentz invariant, etc.) So all you have left is your claims about time dilation, which is self-contradictory and empirically falsified.

    …you'll get the same clock reading on both clocks (20 seconds elapsed) if there isn't any actual time dilation.

    That’s not an answer. The question to you (for the 5th time) is, what does Lorentzian relativity predict? All your subsequent verbiage, trying to evade the question, is wrong, which you’ll understand as soon as you answer the question.

    #1 Hafele-Keating paper. As I've mentioned, they do not actually correct for the clock accuracy deficiencies.

    You’re parroting a thoroughly dishonest and debunked source for your claims. The crude averaging method did indeed show qualitative agreement, and the method based on correcting for the quasi-permanent jumps in drift rates gave improved agreement. Of course, this is of only historical interest, since (as you know) later repetitions with more modern clocks have confirmed the result to high precision. Today it’s possible to measure the time dilation with macroscopic clocks for motions as low as 20 mph (and of course time dilation was already empirically established a hundred years ago).

    Are you referring to Will 1992 "Clock Synchronization and Isotropy..."?

    No, I’m referring to the meaning of “a system of coordinates in which the equations of Newtonian mechanics hold good” in the low speed limit, which entails a definite synchronization based purely on the isotropy of mechanical inertia. We’ve been over this before. If you’re really interested in understanding this, I suggest reading a good book that covers the foundations of special relativity.

    Even though Chou et al 2010 and similar papers seem to be papers testing the reality of grav time dilation, I learned from my dialogue with Stevenson that in fact the experiments published to date don't have the experimental accuracy to find the grav time dilation they claim to find.

    Not true. The papers describing demonstrations of relativistic time dilation with highly accurate clocks do in fact describe demonstrations of relativistic time dilation, and Stevenson’s discussion of practical uses to which a geophysicist might put such clocks was a discussion of just that.

    ReplyDelete
  46. Amos said...
    "
    'first you claim that there was just an easy and clear experiment with clear results.'

    Not true.
    ...
    "

    You are so sure about any of your points. Did you had studied the H&K experiment in detail - or are you just very strong in believe in science honesty?

    I don't believe in the correct interpretation of the H&K experiment, as the Maryland experiment by Alley either. In the report of the Maryland experiment there are some very strange details, and the H&K experiment contains so much uncertainities and sources of error.

    So, the suspicion is strong that they had found what they wanted to found.

    I wonder why are the raw data of these flagship-experiments not on the web, free for everybodys inspection? Perhaps military secret?

    ReplyDelete
  47. Hmmm.....seems like very long discussion..typically if philosophy gets in the way, this happens.

    As John Archibald Wheeler once wrote:

    NO SPACE --> NO TIME!

    In his paper he writes further(I agree almost fully):

    "Fourth and last no: No space, no time. Heaven did not hand down the word "time". Man invented it, perhaps positing hopefully as he did that "Time is Nature's way to keep everything from happening all at once"

    and further:
    "We will not feed time into any deep -reaching account of existence.
    We must derive time — and time only in the continuum idealization — out of it. Likewise with space"

    How simple and how smart!

    And I say: How do you measure time? You can measure the temperature, you can measure the distance between objects. And....how do you measure time? No you cannot...you keep track of the time, no even not that...you keep track of the events...and call it seconds, minutes, years..etc. And even those units(second....to...years) are derived of physical events.
    What is a Earth year? It's a one complete circle of Earth around the Sun :)
    Does time exist? No!

    Cheers!

    ReplyDelete
  48. Metin: "How do you measure time? ... you cannot...you keep track of the time, no even not that...you keep track of the events"

    With respect, "keeping track" is how all measurements are made. Whether keeping track of marks on a measuring stick or events, measurement is "keeping track"

    Saying time does not exist is incoherent; even things that are merely "derived" exist.

    sean s.

    ReplyDelete
  49. Tam Hunt, thanks for the reply. You can probably predict all of my response: a) I have read and enjoyed "Lost In Math" although not the others; b) the problems described in those books are not ones of empirical facts (e.g., no one argues that SUSY or multiverse theories have been supported by empirical evidence) but of philosophy and and of what the most fruitful directions of research will be; c) these are issues where cognitive biases have strong influences, but the whole point of science is to reach agreement on empirical facts, as it has done with respect to Special and General Relativity, i.e., that these theories may be further refinable but are broadly correct and well-supported by empirical data. If some sort of groupthink cognitive biases are instead responsible for their acceptance we might as well give up on science entirely, which I will never do. As badly as I think of the human race, and my guess is that its civilization will collapse within the next 100 years due to over-population/over-consumption and their ecological effects, including global warming, I do not think that badly of our greatest thinkers. If they were and are really that inept, where would that leave me?

    (And there goes another 10-euro soapbox-fine.)

    ReplyDelete
  50. Did you had studied the H&K experiment in detail - or are you just very strong in believe in science honesty?

    I studied it in detail.

    I don't believe in the correct interpretation of the H&K experiment… the H&K experiment contains so much uncertainities and sources of error.

    But you haven’t identified any problem with the data analysis. Your skepticism seems to be based entirely on a thoroughly debunked source, which you accept uncritically. Also, remember that our knowledge of local Lorentz invariance does not rest on Hafele’s cute little airplane stunt, nor even on the later repetitions with more modern clocks. The empirical foundation of special relativity was established long before.

    …of these flagship-experiments…

    They aren’t “flagship experiments”, they are fun demonstrations, mainly to showcase the capabilities of state-of-the-art clocks. Again, the empirical foundation of special relativity was established over 100 years ago. A fun popular demonstration in 1971 of something we already knew for decades is hardly a “flagship experiment”.

    I wonder why are the raw data [are] not on the web, free for everybodys inspection?

    But how would you know they hadn’t falsified the raw data? Fortunately the results have been confirmed by independent groups with much more accurate and stable clocks. It's possible to measure the relativistic time dilation for speeds as low as 20 mph.

    ReplyDelete
  51. Amos and weristdas;

    You two’ve been going on at some length about the reliability of the H&K experiment, and some others.

    I wonder if you two are not missing a point.

    If what I read in this thread is correct, the GPS system is adjusted to compensate for time-dilation; its great precision required that. Assuming that’s correct; is that not—of itself—strong evidence that time-dilation is real; however mysterious that may seem?

    sean s.

    ReplyDelete
  52. @sean s.: see my comments on the Wolf & Petit 1997 paper. All of the papers I've reviewed that purport to show time dilation seem to have serious problems. Wolf & Petit's paper, despite discussing the need for a test theory to examine the invariance of c (isotropic c), and adopting the Mansouri-Sexl test theory, they then forget that GPS satellites are re-synchronized on a regular basis (we must assume) using Einstein synchronization, which assumes invariant c in order to achieve said synchronization.

    We need to acknowledge also that no one on the outside actually knows how the GPS system works due to military secrecy. But it's safe to assume, it seems to me, that the system must include an ongoing Einstein synchronization between GPS satellite clocks and ground-based clocks.

    So the GPS system is putting an invariant c in "by hand" and it is thus to be expected that an analysis of the data would return an invariant c as the result. So it's not that they're adjusting for time dilation as a real physical effect, it's that they're adjusting for the incorrect assumption that c is invariant (c actually behaves like all other interactions, as we'd expect, in that it is indeed affected by the motion of the observer; it's only invariant in the preferred frame of the total field/background frame).

    All of this business about time dilation as a real physical effect (rather than just a coordinate effect akin to changing time zones) comes simply from the choice to use Einstein synchronization, with an assumed invariant c, which leads to the relativity of simultaneity, and then assuming that these choices, made out of convenience for synchronizing distant clocks, are actually indicative of the ontology of the world. But it's not. It's just a convenient way to synchronize clocks and then, based on that synchronization, to be able to translate between inertial frames using the Lorentz transformations. We can recognize and accept the convenience but still reject the ontological implications.

    ReplyDelete
  53. @JimV, have you studied the history of science much? Theories are always changing. There is a general trend toward building on prior theories in a "transcend and include" manner, much as General Relativity attempted to transcend and include Newtonian gravity. But this isn't how it always works. Sometimes theories are simply transcended or rejected. Are we are at this point with supersymmetry, dark matter (as Bee has argued should be the case in her book and recent articles), or general relativity more generally (as Bee has NOT argued but many others have suggested may be the case)? Time will tell. What is clear is that to move forward on many of today's really big physics problems, such as quantum gravity, we need to be ok with questioning sacred cows like quantum theory and GR, and to reconsider these theories from the bottom up. I've stated before my view that in creating a good quantum gravity theory it will likely be GR that has to "give" a lot more than QM does. But we'll see.

    ReplyDelete
  54. TH wrote: It's safe to assume…that the system must include an ongoing Einstein synchronization between GPS satellite clocks and ground-based clocks. So the GPS system is putting an invariant c in "by hand"…

    In normal operation the GPS clocks are indeed re-synched with ground clocks frequently to correct for drift and maintain optimum accuracy, but the baseline rates include corrections for the relativistic effects of gravity and motion, which minimizes the drift between re-synchronizations, and these corrections were validated conclusively when the satellites were first sent into orbit, when the corrections were turned off and before the continual re-synchronization was activated. In this raw state the relativistic effects were clearly confirmed. Moreover, the entire basis of the position triangulation calculations for GPS is that the speed of light is always precisely c (in vacuum) in every direction in terms of the ECI, which of course is varying throughout the year relative to the CMBR. If your beliefs were correct, the GPS triangulation calculations would simply not yield coherent positions. Lastly, please note (again) that "Einstein synchronization" is empirically the same as inertial synchronization.

    We need to acknowledge also that no one on the outside actually knows how the GPS system works due to military secrecy.

    Not true. See Neil Ashby’s detailed explanations and analyses of the GPS system. And of course we can now measure the relativistic effects of half a meter elevation and 20 mph motion in a lab, so your beliefs, in addition to being logically self-contradictory, are abundantly falsified empirically. (It’s interesting that weristdas raised the same conspiratorial concern, i.e., that the military is secretly covering up evidence that special and general relativity are false. Do you guys carpool?)

    … c actually behaves like all other interactions, as we'd expect, in that it is indeed affected by the motion of the observer…

    That misconception has already been debunked. Again, the composition of any two speeds (in terms of inertial coordinates with mechanical synchronization), whether they are speeds of bowling balls or pulses of light, is (u+v)/(1+uv/c^2). Now, it’s certainly true that in terms of other coordinate systems, such as systems related by Galilean transformations, the speed of light depends on the speed of the system, but the laws of physics do not take their usual homogeneous and isotropic form in terms of those systems. As with rotating systems, it is necessary to introduce fictitious forces in order to describe dynamics in terms of those systems. The difference in dynamics shows that relativistic effects are not merely coordinate effects.

    All of this business about time dilation as a real physical effect (rather than just a coordinate effect akin to changing time zones) comes simply from the choice to use Einstein synchronization…

    That misconception (common among students when they first hear about relativity) has already been debunked. Again, according to Lorentzian relativity (which you claim to believe), two physical systems following different paths between two events show different amounts of elapsed time. This is obviously not a coordinate effect.

    … to translate between inertial frames using the Lorentz transformations. We can recognize and accept the convenience but still reject the ontological implications.

    No, your beliefs are self-contradictory, because you claim to believe in Lorentzian relativity but you deny the “ontological implication" of real physical time dilation, which is a prediction of Lorentzian relativity. Again (for the 6th time!), I ask you to tell me what you believe Lorentzian relativity predicts for the elapsed time on an ideal clock moving inertially from (t,x) = (0,0) to (10,5) and from there to (20,0). This requires nothing but grade school arithmetic to answer. Will you answer please? If not, why not?

    ReplyDelete
  55. Tam Hunt, again, super symmetry and dark matter are not empirically established theories; as such they are like the phlogiston theory of combustion prior to Lavoisier's experiments: conceptually possible but not verified. Science proceeds on the basis of empirical observations. That is precisely the point of Dr.s Hossenfelder and Smolin's books, as I see it. Empirical observations agree with SR and GR to high accuracy. Those empirical observations will not go away. Any refined theories will have to explain those same observations.

    Example: according to James Geick's biography of Newton, Newton used his theories of gravity and dynamics to calculate over a thousand astronomical trajectories of planets, moons, and comets, and found them to be in good agreement with observation. Einstein's General Relativity also agrees with those observations, and adds a few more decimal figures of precision. Meanwhile, most engineering calculations outside of NASA and other space agencies still use Newton's methods as being good enough; but if you want to explain the gravity waves detected by LIGO, you need General Relativity.

    Your position seems to be that thousands of scientists and rocket engineers have been miscalculating relativistic time effects as non-linear dilation for about 100 years when it has been a linear coordinate change all along. I don't think anyone who does such math will believe that, and those are the people you would have to convince; with math and empirical data, not conceptual arguments about scientists being wrong in the past (which equally applies to your own views).

    ReplyDelete
  56. Following another link in Dr. Hossenfelder's Twitter log (on naturalness and fine-tuning), I found the following article which summarizes some of the issues in this post and adds some other information:

    http://nautil.us/issue/64/the-unseen/is-it-time-to-get-rid-of-time

    It was interesting to me, so I post it in case others might be interested. The main impression I got from it is that the concept of time needs further scientific development to remove inconsistencies between various successful physical theories; in other words one general concept does not work in all accepted theories, so there are probably further complexities that we need to understand to make progress. (Dr. Rovelli is one of those working on this.) As usual, without knowing the math involved, one only gets hints of what the work consists of.

    ReplyDelete
  57. Tam,

    You make some assumptions about how the GPS system is maintained which I don’t think are safe at all.

    Assuming that the system is updated “by hand” (which seems to mean frequent, ongoing manual updates) is unsafe. Given the precision needed, and the unreliability of manual updates, this is not at all a safe assumption. A GPS system requiring frequent, manual synchronization is simply too risky for the military or anyone else to rely on. One would expect human errors to occur pretty often; have you any evidence of those?

    I’m not at all surprised by a periodic need to resynchronize; Satellite orbits can’t be perfectly elliptical; the Earth’s gravitational field has some observed unevenness, and orbital mechanics tend toward chaotic, so some updates will always be required.

    If the GPS system is using Einstein synchronization assuming an invariant c then the high precision and reliability of that system is strong support for the idea of an invariant c. Resynchronization based on faulty beliefs should lead to regular and significant problems, especially if it’s frequent and manual.

    You are quite critical of what you think are convenient and circular reasoning in others; but if your claims are based on unestablished and unreasonable assumptions, then I cannot see any good reason to agree with your tenuous claims. I think you need to straighten out your own reasoning before critiquing that of others.

    It’s reasonable to regard the evidence as indicating that time-dilation is real and not merely a “coordinate” effect.

    sean s.

    ReplyDelete
  58. @Sean S. You're misunderstanding the phrase "by hand" and it sounds like English is not your first language. It means in this context ad hoc, not done manually. Of course any synchronization is done with very precise instruments. We agree on that. As for your other comments, I urge you read what I wrote a couple more times. And then get a copy of Max Jammer's book Concepts of Simultaneity. The last chapter is on these issues and is all about the "conventionality of simultaneity" debate that centers on the fact that Einstein synchronization and definitions of time and simultaneity are a choice, a convention, that could be different without being "wrong." We've chosen these conventions b/c they work in most ways. But in some ways they don't work and it is in these ways that we need to recognize the conventional choices and reject the ontological implications of those conventional choices.

    ReplyDelete
  59. @Sean S. You're misunderstanding the phrase "by hand" and it sounds like English is not your first language. It means in this context ad hoc, not done manually. Of course any synchronization is done with very precise instruments. We agree on that. As for your other comments, I urge you read what I wrote a couple more times. And then get a copy of Max Jammer's book Concepts of Simultaneity. The last chapter is on these issues and is all about the "conventionality of simultaneity" debate that centers on the fact that Einstein synchronization and definitions of time and simultaneity are a choice, a convention, that could be different without being "wrong." We've chosen these conventions b/c they work in most ways. But in some ways they don't work and it is in these ways that we need to recognize the conventional choices and reject the ontological implications of those conventional choices.

    ReplyDelete
  60. Sean, 5:02 PM, October 14, 2018

    you miss the point: there are two kinds of time dilation. The only questionable point (by my side) is the "kinetic time dilation".

    The GPS system corrects the clocks in respect to the gravitational time dilation. The fractional amount of possible kinetic time dilation goes completely under in the daily standard correction task by earth bounded master control.

    There are several irritations concerning the kinetic time dilation. E.g. gravitation and acceleration are handled in ART as, in much part, related and often similar behavior, let's say Equivalence principle, now I'm asking me, why acceleration (not the movement which results) itself shows no time dilation effect, different from gravitation which shows time dilation effect.

    I think, there is a breach in the equivalence principle since you can decide with clocks experimentally if you are in gravitational field or under acceleration.

    ReplyDelete
  61. I don't recall that the relativistic doppler effect for light (e.g., red shift) has been mentioned in this thread, so I'll bring it up. It was derived using (kinetic) time-dilation. As far as I know all astronomical data uses this effect with no inconsistencies noted. I would think any university physics lab could design experiments to test it and probably many of them have. (It predicts that the frequency of a following light ray will go to zero in a non-linear fashion as the speed of the measuring device increases to the c-limit.)

    As I understand it, Tam Hunt will agree that this effect exists but claim it is not a real time dilation, at which point I don't understand how a measured, non-linear effect is not real.

    ReplyDelete
  62. Tam wrote: …get a copy of Max Jammer's book Concepts of Simultaneity. The last chapter is on these issues and is all about the "conventionality of simultaneity" debate that centers on the fact that Einstein synchronization and definitions of time and simultaneity are a choice, a convention…

    You overlooked the discussion at the end of the chapter, noting that the synchronization of inertial coordinate systems is uniquely determined by the requirement for the dynamical equations to take their homogeneous and isotropic form, which after all is the definition of inertial coordinates. Naturally we are free to use non-inertial coordinate systems (e.g., with different synchronizations) if we like, but then we must introduce fictitious force terms to the dynamical equations, and those terms are nothing but the corrections necessary to account for the use of non-inertial coordinates. This was all spelled out explicitly for the philosophers in the 1990s, and has always been implicitly understood by physicists. As Einstein said regarding the adoption of the Lorentz transformations, “With the given physical interpretation of coordinates and time, this is by no means merely a conventional step, but implies certain hypotheses concerning the actual behavior of moving measuring-rods and clocks, which can be experimentally validated or disproved”.

    But in some ways they don't work…

    You’ve not cited any ways in which special relativity “doesn’t work”. At first you mistakenly thought the CMBR frame and quantum entanglement conflict with special relativity, but it’s been explained that Lorentz invariance is local not global, and quantum field theory is explicitly Lorentz invariant. Your only remaining example is time dilation, which you claim does not exist, despite the overwhelming experimental evidence. (The errors in your critiques of the experimental results have been thoroughly explained.)

    Moreover, you claim that Lorentzian relativity (which you espouse) does not predict any real physical time dilation, but that’s wrong. To clear this up, I’ve asked you repeatedly to say what Lorentizan relativity predicts for the elapsed time on clocks in a very simple example. You refuse to answer, or even to acknowledge the question. What should we conclude from this?

    and it is in these ways that we need to recognize the conventional choices and reject the ontological implications of those conventional choices.

    But the “ontological implications”, such as real physical time dilation (and all the other confirmations of local Lorentz invariance) are empirically confirmed, so rejecting them would be irrational.

    weristdas said: I'm asking me, why acceleration (not the movement which results) itself shows no time dilation effect, different from gravitation which shows time dilation effect. I think, there is a breach in the equivalence principle…

    There’s no breach in the equivalence principle. Think about a clock at the rim of a spinning centrifuge. If you evaluate this in terms of inertial coordinates (say, the hub frame), there is no gravitational field and there is time dilation due to the speed of the clock, but if we evaluate this in a rotating system of coordinates in which the clock is stationary, there is a pseudo-gravitational field that results in exactly the same time dilation. What you're missing is that the time dilation due to acceleration/gravity is not just a function of the difference in accelerations, it is a function of the difference in potential energy, i.e., the amount of work that would be required to “lift” a clock from one location to the other, so it depends on the distance as well as the magnitude of the acceleration.

    ReplyDelete
  63. Tam;

    I’ve been reading your exchange with Amos and I have to say I’m finding him more persuasive than you.

    I might be making an error in that, which would not be my first even if you only considered today. But there it is.

    You really lost me at “You're misunderstanding the phrase ‘by hand’ and it sounds like English is not your first language. It means ...

    Not only is English my first language, English is my ONLY language. I have failed to learn Latin, Spanish, French, Arabic, and Irish. I used to know a little Dutch, but that’s all gone. I am a polyglot flop.

    Do programming languages count?

    < humor >

    Q: What do you call someone who speaks three languages? A: Trilingual.
    Q: What do you call someone who speaks two languages? A: Bilingual.
    Q: What do you call someone who speaks just one language? A: American.

    < /humor >

    I am thoroughly American (I grew up on the Great Plains); but I’m good enough at English to get an engineering degree and graduate from law school. For most English speakers, “by hand” implies “manually”. Ad hoc means something else.

    As Amos asked; I shall ask also: you wrote that “Einstein synchronization and definitions of time and simultaneity are a choice, a convention, that could be different without being "wrong." We've chosen these conventions b/c they work in most ways. But in some ways they don't work. ” (emphasis added) (I still try to follow the Blue Book markup rules as appropriate).

    So: pray tell-can you cite examples where “they don’t work”?

    sean s.

    ReplyDelete
  64. weristdas;

    Perhaps I’m naïve (more likely than not) but kinetic time dilation always seemed less mysterious to me than gravitational time dilation.

    E.g. gravitation and acceleration are handled in ART as, in much part, related and often similar behavior, let's say Equivalence principle, now I'm asking me, why acceleration (not the movement which results) itself shows no time dilation effect, different from gravitation which shows time dilation effect.

    OK, I admit it: I cannot parse this sentence. Is ART capitalized because it’s an acronym? or because you’re emphasizing it?

    I could attempt to figure this out, but I’d rather that you clarify it. If you’d be so kind.

    sean s.

    ReplyDelete
  65. @Amos, responding to your last comment below. I've been working up responses to your last two lengthy responses but here's the crux of where we've been disagreeing to date below. It actually seems like we now agree on where the miscommunication and broader disagreement lies. You've generally just been assuming the principle of relativity (as described by Einstein in his book Relativity: The Special and the General Theory), when I've been arguing that the principle of relativity is indeed a postulate, of course, and a convention, not a necessary choice. I go further and suggest that it's actually not a good choice in terms of leading to ontological conclusions about the nature of time or the universe more generally.

    "Again, the composition of any two speeds (in terms of inertial coordinates with mechanical synchronization), whether they are speeds of bowling balls or pulses of light, is (u+v)/(1+uv/c^2). Now, it’s certainly true that in terms of other coordinate systems, such as systems related by Galilean transformations, the speed of light depends on the speed of the system, but the laws of physics do not take their usual homogeneous and isotropic form in terms of those systems. As with rotating systems, it is necessary to introduce fictitious forces in order to describe dynamics in terms of those systems. The difference in dynamics shows that relativistic effects are not merely coordinate effects."

    This is not true. Why are you using the speed of light in your equation? That is only included b/c you're used to it being included in relativity theory. There is no a priori reason that normal speeds bear any relation to the speed of light, or that the speed of light is a cosmic speed limit. Lorentz and others, and eventually Einstein, as I previously described, looked to the speed of light as an upper speed limit, in terms of limiting applicability of the theory, before it was deemed to be ontologically significant. As you acknowledge, Galilean transformations do NOT rely on the speed of light for translations. You write: "The laws of physics do not take their usual homogeneous and isotropic form in terms of those [Galilean] systems." You are proving my point, of course. You've simply been assuming the truth of the point you've asserted throughout our discussion. And I've been trying time and time again to show you that you are begging the question. My entire point here has been to argue that the principle of relativity is a postulate of SR that we need to reconsider. It looks like we now have at least established that point. You can't state postulates of a theory as attempted support for that same theory. That, again, is pure tautology.

    In terms of rotating systems, I think you have it backwards. As this quote from Einstein (in a 1920 address at Leiden Univ. where Lorentz was based) himself makes clear, we need to posit a structure to space (ether or whatever you prefer to call it) in order to explain acceleration and rotation:

    "To deny ether is ultimately to assume that empty space has no physical qualities whatever. The fundamental facts of mechanics do not harmonize with this view... Besides observable objects, another thing, which is not perceptible, must be looked upon as real, to enable acceleration or rotation to be looked upon as something real ... The conception of the ether has again acquired an intelligible content, although this content differs widely from that of the ether of the mechanical wave theory of light ... According to the general theory of relativity, space is endowed with physical qualities; in this sense, there exists an ether. Space without ether is unthinkable; for in such space there not only would be no propagation of light, but also no possibility of existence for standards of space and time (measuring- rods and clocks), nor therefore any spacetime intervals in the physical sense."

    ReplyDelete
  66. My fault.

    ART is the German abbreviation for General Relativity Theory (Allgemeine RelativitätsTheorie)

    ReplyDelete
  67. Further to my comment on the relativistic doppler effect, the Wikipedia article on it includes this:

    "The transverse Doppler effect and the kinematic time dilation of special relativity are closely related. All validations of TDE represent validations of kinematic time dilation, and most validations of kinematic time dilation have also represented validations of TDE. An online resource, "What is the experimental basis of Special Relativity?" has documented, with brief commentary, many of the tests that, over the years, have been used to validate various aspects of special relativity.[16] Kaivola et al. (1985)[p 13] and McGowan et al. (1993)[p 14] are examples of experiments classified in this resource as time dilation experiments. These two also represent tests of TDE. These experiments compared the frequency of two lasers, one locked to the frequency of a neon atom transition in a fast beam, the other locked to the same transition in thermal neon. The 1993 version of the experiment verified time dilation, and hence TDE, to an accuracy of 2.3×10-6."

    ReplyDelete
  68. Tam wrote: You've generally just been assuming the principle of relativity…

    No, we don’t “just assume” the principle of relativity. It is established by countless observations and measurements, just like other fundamental physical principles such as local conservation of energy and momentum. We do not “just assume” these principles, willy-nilly. They are the extracted essence of our most well-established empirical knowledge.

    I've been arguing that the principle of relativity is indeed a postulate, of course, and a convention, not a necessary choice. I go further and suggest that it's actually not a good choice…

    I know that’s what you’re arguing, and I’ve explained thoroughly why you are wrong. You offer no answer to those explanations. Also, you steadfastly refuse to answer even the simplest question about your beliefs, e.g., what does Lorentzian relativity predict for the elapsed time for the two clocks. The answer is obviously not a matter of convention or choice, the two clocks will show some definite amounts of elapsed times. Why will not you answer this?

    I explained that the composition of any two speeds in terms of inertial coordinates with inertial synchronization is (u+v)/(1+uv/c^2). In response you say

    This is not true. Why are you using the speed of light in your equation?

    Sorry, but it is true (for special relativity and for Lorentzian relativity and as an empirical fact). The constant value denoted by “c” fundamentally represents the constant of proportionality between inertial mass and energy. Newton thought a material particle subjected to a constant force would undergo constant acceleration, but now we know that the energy imparted to accelerate a particle increases it’s inertia (resistance to acceleration), and the constant of proportionality is 1/c^2 (so small that Newton didn’t notice it). The energy and hence the inertia of the particle increase with speed, and they approach infinity as the particle approaches the speed c. (Newton thought the inertia didn’t increase at all, and the energy would approach infinity only as the speed approached infinity.)

    The fundamental relation for any entity, whether it is a baseball or a photon, is (mc^2) = E^2 – (pc)^2 where m is the rest mass, E is the energy, and p is the momentum. For a stationary object we have p=0 and hence E=mc^2. For a photon we have m=0 and hence p=E/c. These relations imply that inertial coordinate systems are related by Lorentz transformations, and that massless energy necessarily propagates at the speed c in terms of such coordinates. None of this is conventional or “just assumed”. These are all empirical facts, facts that Newton didn’t know, with enormous physical consequences.

    You write: "The laws of physics do not take their usual homogeneous and isotropic form in terms of those [Galilean] systems." You are proving my point, of course… That, again, is pure tautology.

    No, you misunderstand. Again, we can physically (empirically) establish coordinate systems in which the equations of physics are homogeneous and isotropic, and these are called inertial coordinate systems, and they are empirically related by Lorentz transformations. None of this is conventional or tautological. The inertia of energy is a palpable physical fact, and all the physical consequences of special relativity derive from it.

    As this quote from Einstein himself makes clear…

    What Einstein said was correct, but he contradicts (not supports) your anti-relativity beliefs. Semantics aside, his whole point is that the “ether” of spacetime is relativistic. Look, all you’re doing is reciting the most common anti-relativity fallacies. Instead of posting well-known quotes that you thoroughly misunderstand, could you please just tell me what you think Lorentzian relativity predicts for the elapsed times on those two clocks? Is the answer a matter of choice or convention? Do you deny that energy has inertia? Do you deny that (mc^2) = E^2 – (pc)^2 ?

    ReplyDelete
  69. Do you deny that (mc^2) = E^2 – (pc)^2 ?

    Typo... should be (mc^2)^2 = E^2 – (pc)^2.

    ReplyDelete
  70. Amos, 2:20 PM, October 17, 2018

    I wonder why the answer to my question concerning time dilation by acceleration usually leads to answers with circular motion examples. Why that complication?

    I have discussed that aspect elsewhere, and the advocates of relativity theory had stated, that it is possible to speak about "acceleration time dilation" analogue to "gravitational time dilation" (think of the equivalence principle).

    For German speakers:

    https://scilogs.spektrum.de/quantenwelt/mit-dem-dopplereffekt-relativitaet-durchschauen/

    Albrecht Storz 11. Oktober 2018 @ 14:52 and following posts

    ReplyDelete
  71. I wonder why the answer to my question concerning time dilation by acceleration usually leads to answers with circular motion examples. Why that complication?

    Examples can be given for paths of any shape, not just circles. It so happens that circular paths are simple to describe because the distance is constant (strange that you regard them as a “complication”), but if you prefer, we can just as well describe examples for linear motion, such as the usual “away and back” twins scenario. For this or any other path we always get exactly the same results, whether we just integrate sqrt(dt^2 – dx^2 – dy^2 – dz^2) in terms of any inertial coordinate system t,x,y,z with c=1 or we use an accelerating coordinate system and account for the pseudo-gravitational fields (difference in potential energy) in terms of those coordinates.

    … the advocates of relativity theory had stated, that it is possible to speak about "acceleration time dilation" analogue to "gravitational time dilation" (think of the equivalence principle).

    Yes, when we treat acceleration as a pseudo-gravitational field, we get the same results as we would get in a corresponding real gravitational field for the same difference in potential energy. Of course, experiments in flat spacetime don’t show this equivalence. To demonstrate the equivalence principle it’s necessary to examine how things behave in a real gravitational field, and compare with how they behave in a pseudo-gravitational field (accelerated system in flat spacetime). Many genuine tests of the equivalence principle have been performed, and it has always been confirmed, but this is sort of irrelevant to your denial of special relativity within its domain of applicability based on the operational definition of inertial coordinates, and the fact that different choices for coordinates can’t affect the values of any observables.

    ReplyDelete
  72. Amos,

    "Yes, when we treat acceleration as a pseudo-gravitational field, we get the same results as we would get in a corresponding real gravitational field for the same difference in potential energy."

    I wonder why there should be a difference by the way we "treat" the case, or look at the case.

    Question: If we observe a displacement of an object in the first case at a constant speed, in the second case with accelerating speed, in both cases with the same displacement in the same time, is in both cases the time dilation amount the same, and if yes, why does the acceleration not contribute a supplementary time dilation effect?

    Think of the additional effect of gravitational and kinetic time dilation in the case of the Hafele & Keating experiment.

    ReplyDelete
  73. weristdas said I wonder why there should be a difference by the way we "treat" the case, or look at the case.

    You misread. The sentence you quoted was “When we treat acceleration as a pseudo-gravitational field, we get the same results as we would get in a corresponding real gravitational field for the same difference in potential energy.” (This is just the equivalence principle.) So it doesn't make sense to "wonder why there should be a difference".

    Question: If we observe a displacement of an object in the first case at a constant speed, in the second case with accelerating speed, in both cases with the same displacement in the same time, is in both cases the time dilation amount the same..

    No, the elapsed times for two paths between two given events, one path accelerated and the other not, are generally different, depending on the speeds and gravitational potentials along the paths.

    Why does the acceleration not contribute a supplementary time dilation effect?

    In every case, the elapsed time along any path is simply the integral of sqrt(g_mn x^m x^n) along that path, regardless of whether the path is accelerating or not. The metric coefficients (g_mn) depend on the choice of coordinate systems in such a way that the integral is invariant. For example, in terms of an inertial coordinate system in flat spacetime we have the Minkowski metric coefficients, and time dilation (compared with coordinate time) of an accelerating object is purely a function of speed in those coordinates. As an alternative, we can select an accelerating coordinate system, in which the object is stationary, but the metric coefficients are correspondingly different, yielding the same time dilation as before. (By the equivalence principle this matches the time dilation for a stationary object in the equivalent gravitational field as explained above, but it isn’t necessary to know this in order to compute the time dilation in this situation in flat spacetime.)

    The standard anti-relativity fallacy is that the object should exhibit double time dilation, once for its speed in inertial coordinates and once for the metric effect in accelerating coordinates (in which it is stationary). Needless to say, that’s absurd, because these are the same effect, just described in two different coordinate systems. (If you are 6 feet tall in terms of one system of coordinates, and 6 feet tall in terms of another system of coordinates, this doesn’t mean you are really 12 feet tall.)

    Think of the additional effect of gravitational and kinetic time dilation in the case of the Hafele & Keating experiment.

    In that situation there are two distinct physical effects, one due to motion and one due to the Earth’s gravitational field. These effects are both present in terms of a single system of coordinates such as the Schwarzschild coordinates with the corresponding metric coefficients, and, as always, the elapsed time on each clock is simply the integral of sqrt(g_mn x^m x^n) along the path of that clock. We do not add the time dilations computed for multiple different coordinate systems, we compute the time dilation for a single coordinate system. It can be any system we choose; the answer is the same.

    ReplyDelete
  74. Amos,

    I can't understand, why gravitation and motion produces two different measurable and distinct effects, but acceleration and motion not. As I had thought, a clock above another clock in an accelerated system should show time dilation, compared with the lower clock, as in the case of two clocks in gravitational field and constant motion.

    But okay, you are totally sure about Einsteinian theory.

    I'm not. The system may be consistent, seems to be consistent. But I don't see really convicting experiments which were performed undoubtedly and with unambiguous results.

    Consistency doesn't necessary imply reality.

    ReplyDelete
  75. weristdas said I can't understand, why gravitation and motion produces two different measurable and distinct effects, but acceleration and motion not.

    You’re confusing accelerating objects and accelerating coordinate systems. Again, the elapsed proper time along any path is the integral of sqrt(g_mn x^m x^n) along that path, where the x^m are any system of coordinates (within the relevant diffeomorphism class). This answers all your questions. For example, in terms of an inertial coordinate system there are no differences in potential energy entailed in the g_mn, so time dilation for an accelerating object is due purely to speed, but we can consider the same object in terms of an accelerating coordinate system (in which the object is stationary), and in these terms the object has no speed or acceleration (derivatives of position with respect to time), but the g_mn entail differences in potential energy. In either case, when we integrate sqrt(g_mn x^m x^n) along any path we get exactly the same results. We don’t add these results together, they are the same result, merely described in terms of different coordinates.

    Now, the equivalence principle asserts that the equations of physics in terms of the accelerating coordinates (e.g., in which the object is stationary) in the absence of gravity corresponds locally to the equations in terms of a stationary coordinate system in a suitable gravitational field. This is encoded in the g_mn for a stationary coordinate system in a gravitational field, representing an additional effect when we integrate sqrt(g_mn x^m x^n) along a path.

    A clock above another clock in an accelerated system should show time dilation, compared with the lower clock, as in the case of two clocks in gravitational field and constant motion.

    Right, and it does.

    But okay, you are totally sure about Einsteinian theory. I'm not.

    Any theory can be doubted, but all your concerns about relativity seem to be based on misunderstandings and misinformation.

    The system may be consistent, seems to be consistent.

    Is consistent.

    Consistency doesn't necessary imply reality.

    Right, that’s why we conduct tests and observations. Special and general relativity have been subjected to many empirical tests (within the regime you consider), and no violations have been found. For example, the most accurate and stable clocks can detect the time dilation due to just 20 mph and half a meter of elevation.

    I don't see really convicting experiments which were performed undoubtedly and with unambiguous results.

    Without understanding the theory you can’t really assess the tests of it. There have been countless tests and demonstrations of special and general relativity during that past century, including many recent tests with very high precision, and so far no violations have been seen. This is not ambiguous.

    ReplyDelete
  76. I think, now I'm understanding Tam Hunt's concerns.

    An observer who is moving with approximately light speed should observe the same universe that we observe. But his universe is e.g. only 300 million years old. That's impossible in the view of the necessary time of development. (Implying "big bang theory")

    In particular, if natural constants as electric charge (electron, proton, ...) or Feinstrukturkonstante, etc. are invariant.

    ReplyDelete
  77. Amos,

    "and half a meter of elevation"

    I've never questioned graviatational time dilation.

    ReplyDelete
  78. I haven't followed your discussion but an observer moving at (or near) the speed of light relative to us would of course not observe the universe the same way as we do because the universe is not Lorentz-invariant. It has a preferred frame.

    ReplyDelete
  79. I think, now I'm understanding Tam Hunt's concerns. An observer who is moving with approximately light speed should observe the same universe that we observe...

    No, Tam’s concerns were already answered (several times). Again, local Lorentz invariance is local, not global. For example, we can discern our speed relative to the frame in which the CMBR is isotropic. This is perfectly consistent with local Lorentz invariance (special relativity), and of course with general relativity. Notice that it isn’t even necessary to consider the whole universe to observe violations of global Lorentz invariance… any gravitational field violates local Lorentz invariance, which is why general relativity was developed. But in sufficiently small regions of spacetime over which gravitational potential is fairly constant, special relativity applies. That’s what local Lorentz invariance means.

    "and half a meter of elevation" … I've never questioned graviatational time dilation.

    You overlooked the detection of time dilation for speeds as low as 20 mph at constant elevation. This, along with countless other precise demonstrations of local Lorentz invariance over the past 100 years, conclusively falsifies your beliefs.

    Again, the elapsed proper time along any path is the integral of sqrt(g_mn x^m x^n) along the path. This embodies both local Lorentz invariance (special relativity) and the effects of gravitation (general relativity). What part of this do you disagree with?

    ReplyDelete
  80. Amos,

    thanks for the great job you are doing here to clarify SR and GR. I still wonder why Tam is not at all reacting to the obvious 2*sqrt(10²-5²)=17.32 or (mc²)² = E² – (pc)².

    You just had a minor typo, it should be: … any gravitational field violates (global) Lorentz invariance …

    Maybe this also might help:
    - The symmetries of GR are local Lorentz and diffeomorphism invariance.
    - (global) Lorentz invariance in SR becomes local Lorentz invariance in GR.
    (analog to: global phase invariance in QM becomes local gauge invariance in QFT)
    - Lorentz invariance/symmetry refers to the laws, not to the actual matter/radiation distribution.
    - equivalence principle is not a symmetry, it relates two physical situations (with and without gravity or acceleration)
    - here and here are tons of experiments that confirm SR and GR.

    ReplyDelete
  81. Amos,

    "You overlooked the detection of time dilation for speeds as low as 20 mph "

    I'm sure we are able to detect the Doppler effect at 20 mph.

    ReplyDelete
  82. Reimond said You just had a minor typo, it should be: … any gravitational field violates (global) Lorentz invariance …

    Yes, thanks.

    weristdas, in response to my statement "You overlooked the detection of time dilation for speeds as low as 20 mph", replied I'm sure we are able to detect the Doppler effect at 20 mph.

    Sure, we can detect the first-order Doppler effect of 20 mph with a cheap hand-held radar speed gun used by traffic police and baseball coaches. On the other hand, detection of time dilation, which is a second-order effect, for just 20 mph requires a state-of-the-art atomic clock. This has actually been done, so your denial of relativistic time dilation has been conclusively falsified (again). Clearly your denial is just based on misunderstanding, because you’re confusing the Doppler effect with time dilation.

    ReplyDelete
  83. I've been reading about the latest ytterbium atomic clocks, which have unprecedented accuracy and, importantly, reproducibility (which allows two clocks to be compared usefully). These clocks can detect the difference -- due to GR -- of just one centimeter difference in altitude. That's impressive!

    https://www.nist.gov/news-events/news/2018/11/nist-atomic-clocks-now-keep-time-well-enough-improve-models-earth

    While I'm here: I never did get an answer from time dilation deniers about how muons created by cosmic ray collisions high in the atmosphere manage to reach the surface of the Earth without time dilation. Which they definitely do.

    ReplyDelete
  84. Chris, this is a good example of how easy it is to misread papers. The new paper on ytterbium clocks says nothing about actual tests of GR. Rather, it discusses new clocks that are more accurate than today,s best clocks, that could in theory be used to test gravitational fields more accurately -- under the assumption that gravitational fields do in fact lead to time dilation. But no such tests have taken place yet and won't be ready for a number of years. And even if those tests could take place they're built on a tautological assumption that any measured changes in clock rates at different locations would be due to gravitational field differences. But there are many other things that could lead to such changes. And gravitational fields are always changing slightly on our planet so you couldn't even set up a series of repeated experiments to test whether observed differences in clock rates were due to specific grav field differences in different locations.

    ReplyDelete
  85. As for muons and time dilation, it's more sensible to explain the data as interaction with the grav field, and physical slowing of processes, rather than time dilation. Bethell explains this well in his book Questioning Einstein as follows:

    'Actually, another plausible explanation was suggested— inadvertently. George Gamow himself was responsible. As he was a friend of Einstein’s and an orthodox relativist, we can be sure that he intended no heresy. But he said quite plainly what surely does happen with the fast-moving mesons. His error (I mean from the relativists’ point of view) only shows how thin is the barrier separating the counter-intuitive world of relativity physics from a more straightforward way of looking at the world. Gamow said:

    "Slowing down of all physical processes in fast-moving systems was observed directly in the case of the decay of mesons, the unstable elementary particles which constitute an essential fraction of cosmic rays coming down to the surface of the earth at extremely high speed."

    It surely is the physical processes that slow down, not “time.” The process of decay within the atomic particle as it flies through the Earth’s gravitational field is slowed down by its transit through that medium.'

    Bethell, Tom. Questioning Einstein: Is Relativity Necessary? (Kindle Locations 3478-3487). Vales Lake Publishing, LLC. Kindle Edition.

    ReplyDelete
  86. Tam Hunt: “As for muons and time dilation, it's more sensible to explain the data as interaction with the grav field, and physical slowing of processes, rather than time dilation.

    ‘It surely is the physical processes that slow down, not “time.” The process of decay within the atomic particle as it flies through the Earth’s gravitational field is slowed down by its transit through that medium.'” [quoting Bethell quoting Gamow]

    If all processes are slowed, that is essentially what time dilation means. Slowed processes reveal slowed time: time dilation.

    Slowed physical processes due to interaction with the gravity field is indistinguishable from time dilation due to interaction with the gravity field. So there’s no reason to object to one if you accept the other.

    sean s.

    ReplyDelete
  87. It surely is the physical processes that slow down, not “time.”

    But then physical processes are the only things we measure when we claim to measure "time". In other words, Gamow's statement is consistent with the view that time is just a relational concept, not something in and of itself.

    ReplyDelete
  88. bud rap; “In other words, Gamow's statement is consistent with the view that time is just a relational concept, not something in and of itself.

    Gamow’s statement is consistent with the idea that time is something in and of itself; Gamow’s statement implies a singular, universal time “flow” instead of one subject to local conditions.

    Your notion of “relational time” is something you’ve commented about in the past, but you have yet to provide any empirical evidence showing time being derived from other phenomena, or provide examples of “change without time”.

    I do understand your desire to see direct, empirical data re. time; that would be the “gold standard” would it not? I’d love to have some too.

    In the absence of that data--and our utter inability to even propose an experiment that might produce that “gold standard” data--rational, science-minded folks don’t just decide that time is derivative, emergent, composite, or caused by other phenomena. Where is the “gold standard” evidence for any of that? Nowhere.

    We all experience time; we never experience time being “made”.

    Claiming time is created by change is like claiming gravity is created by things falling down.

    sean s.

    ReplyDelete
  89. @Tam Hunt: You may have misread my comment; I didn't mention any "actual tests." You are correct we assume time dilation occurs due to gravity, but that assumption seems correct. (If it isn't, GPS has a lot of 'splaining to do!)

    To be clear, the ytterbium clocks just provide better accuracy on tests already made, repeatedly, of GR. (The GPS system tests it on a moment-by-moment basis!) BTW: Multiple clocks at the same location, but at different elevations, remove objections about locations varying.

    Serious people have tested this in very serious ways. Demonstrating GR incorrect due to some other theory requires very serious proof.

    As for the muons, how is it that atomic constants, like the frequency at which atoms vibrate, or the half-life of particle decay, change due flying "through Earth's gravitational field."

    And what about fast-flying muons outside a gravity field? Does their half-life change?

    Given the consistency of c, time dilation of frame moving frame with regard to another is a geometric necessity, so denying it requires disputing the consistency of c (or of basic geometry). I accept that consistency as axiomatic.

    @bud rap: But then physical processes are the only things we measure when we claim to measure "time"

    The idea time is relational goes back at least to Leibniz, and seems (to me) to be an ontological metaphysics issue (and, for the record, I side with Kant's reply to Leibniz that time is fundamental).

    The thing about ontological questions is that, at some point, something just is; it's not turtles all the way down. Something is fundamental.

    I see a relation as complex and comprised of fundamental parts, and to my analysis, a time relation, as Kant said, necessarily assumes time as the context of the relation. You can't construct time from time relations, but you can construct time relations from time.

    ReplyDelete
  90. Sean S., not so. Does time dilate when an ice breaker travels through ice rather than open ocean? Does time dilate for a spoon when you pass it through molasses rather than air? The point is that the passage of time more generally proceeds exactly the same, but certain physical processes may be slowed. Does such slowing mean time dilates from the perspective of the muon? Considering the larger context it seems far more sensible to me to explain the data without reference to time dilation.

    ReplyDelete
  91. @sean s,

    "...you have yet to provide any empirical evidence showing time being derived from other phenomena, or provide examples of “change without time”."

    I do not have to provide evidence for the non-existence of something that cannot be empirically demonstrated to exist. On the contrary, it is upon you, since you propose the existence of time, that the burden of proof falls. I do not have to provide examples of "change without time", it is you who has to provide examples of "time without change" - but by your own admission, you cannot do that.

    That gets us to this peculiar straw-man argument from authority:

    "In the absence of that data--and our utter inability to even propose an experiment...rational, science-minded folks don’t just decide that time is derivative, emergent, composite, or caused by other phenomena."

    I have no idea who these "rational, science-minded folks" are, but from the context, it would appear they are just those "folks" who agree with you. Argument from authority is weak to begin with, diluted like that, it is just solipsistic nonsense.

    As to the straw-man portion, at no point did I say or imply that time "is derivative, emergent, composite, or caused by other phenomena." I have said only that it is a human concept that has no scientifically meaningful, substantival existence - that it does not exist, except subjectively, in the space between the ears of those human folk who choose to believe in its existence.

    @Chris Sonnack

    "The thing about ontological questions is that, at some point, something just is; it's not turtles all the way down. Something is fundamental."

    There is a distinction to be made here between the philosophical arguments of Leibniz and Kant and the scientific evaluation of same. The philosophical argument has and will go on incessantly; it reduces to a matter of opinion.

    In the context of science, however, the resolution is straightforward. There is no empirical evidence for the existence of a substantival time - none. So from a scientific perspective, the relational interpretation is in agreement with the non-observation of a substantival time, and it therefore follows that time cannot be fundamental.

    "You can't construct time from time relations, but you can construct time relations from time."

    First of all, that is a circular argument. Second, there is no scientific need to logically "construct" something that cannot, in fact, be demonstrated to exist.

    ReplyDelete
  92. Tam Hunt:

    Does time dilate when an ice breaker travels through ice rather than open ocean? Does time dilate for a spoon when you pass it through molasses rather than air? ... passage of time more generally proceeds exactly the same, but certain physical processes may be slowed.

    The breakdown of muons created by cosmic rays is a process that is categorically different from the motion of a ship or a spoon through molassas. Motion (translation through space) is a categorically different “process” from muon decay with categorically different considerations; we were commenting on the latter, not the former.

    I stand by my comment: slowed physical processes due to interaction with the gravity field is indistinguishable from time dilation due to interaction with the gravity field. So there’s no reason to object to one if you accept the other.

    sean s.

    ReplyDelete
  93. @bud rap: "There is no empirical evidence for the existence of a substantival time - none."

    What empirical evidence is there for relational time? And what do you consider empirical evidence?

    "So from a scientific perspective, the relational interpretation is in agreement with the non-observation of a substantival time, and it therefore follows that time cannot be fundamental."

    Are you suggesting, because we haven't (yet!) directly observed something, it cannot be fundamental? Earlier in history, was the photon not fundamental? (Maybe I should first ask what empirical evidence you accept regarding photons?)

    In point of fact, no one knows what time is. Not me, not you, not anyone. This is all opinion.

    "First of all, that is a circular argument. Second,..."

    There is nothing circular about: B derives from A, but A doesn't derive from B. (In fact, it's explicitly not circular.)

    Secondly, you may have missed the point. It is trivial to create relations in time, but how does one create time given only the idea of a time relation?

    To me a relation is an operator on two things that (necessarily) already exist. The relation doesn't bring the related things into existence, so how does that work?

    What exactly is a time relation that it is more fundamental than time? Can you define one without necessarily implying time?

    A characteristic of fundamental things is that, because they lack parts, any properties they have necessarily apply to the whole. But time relations do have parts and properties that apply only to those parts. That seems not fundamental to me. That time relations even have parts seems not fundamental.

    The time that passed while I wrote this response: How many relations are involved? Are there relations between these moments and every moment that's passed since the big bang? Why or why not? What creates a relation? (Is there an "observer" necessary?) How many relations exist for a single moment in time? Can relations be destroyed?

    I find questions like these suggestive that time relations can't be fundamental, but obviously your mileage varies. Maybe you have good answers to these questions, but for now I'll put my money on time being a thing that simply is. (I'd comfortably put money on it being more fundamental than space.)

    ReplyDelete
  94. bud rap;

    ... it is upon you, since you propose the existence of time, that the burden of proof falls.

    So now you’re challenging the very existence of time, a phenomena everyone experiences and which has been the subject of thought and discussion since the beginning of history?

    That gets us to this peculiar straw-man argument from authority: ...

    Clearly you do not know what either straw-man argument or argument from authority mean.

    ... at no point did I say or imply that time ‘is derivative, emergent, composite, or caused by other phenomena.’ I have said only that it is a human concept that has no scientifically meaningful, substantival existence - that it does not exist, except subjectively, in the space between the ears of those human folk who choose to believe in its existence.

    Interesting. In two sentences you deny that time is created by other phenomena and then say that time IS created by other phenomena. Even if time is an illusion, it would exist, and be created by other phenomena (the mind).

    Way back in August you wrote that “Time is just such a human concept derived from the observation of physical processes.

    Your statement from August implies that time exists, and is caused by the human mind. That’s what “relational time” means.

    Since August, you have manytimes written that you think time is just a relational concept, not something in and of itself; you reiterated that idea just this week.

    Now you deny all you wrote????

    Hmm.

    There is no empirical evidence for the existence of a substantival time - none. So from a scientific perspective, the relational interpretation is in agreement with the non-observation of a substantival time, and it therefore follows that time cannot be fundamental.

    There is no empirical evidence for the existence of relational time; none. So, from a scientific perspective, the substantival interpretation agrees with the non-observation of relational time. It therefore follows that time must be fundamental.

    Time without change is stasis; change without time is absurd.

    sean s.

    ReplyDelete

COMMENTS ON THIS BLOG ARE PERMANENTLY CLOSED. You can join the discussion on Patreon.

Note: Only a member of this blog may post a comment.